You are on page 1of 55

curso de post-grado para profesores

especialidad en matemática
COMBINATORIA
Equipo de Diseño:
Carlos Mauricio Canjura Linares
Claudia Patricia Corcio
Ernesto Américo Hidalgo Castellanos
Humberto Alfonso Sermeño Villalta
Eder Alexander Jacobo Arévalo
Aarón Ernesto Ramı́rez Flores
28 de junio de 2010

1
ÍNDICE ÍNDICE

Índice
1. Principios de Conteo 4
1.1. PRINCIPIO DE LA SUMA . . . . . . . . . . . . . . . . . . . . . . . . . . . . . . . 4
1.2. PRINCIPIO DE LA MULTIPLICACIÓN . . . . . . . . . . . . . . . . . . . . . . . . 5
1.3. PRINCIPIO DE BIYECCIÓN . . . . . . . . . . . . . . . . . . . . . . . . . . . . . . 6
1.4. INCLUSIÓN-EXCLUSIÓN . . . . . . . . . . . . . . . . . . . . . . . . . . . . . . . . 9
1.5. RECURRENCIA. . . . . . . . . . . . . . . . . . . . . . . . . . . . . . . . . . . . . . 10
1.6. INDUCCIÓN MATEMÁTICA. . . . . . . . . . . . . . . . . . . . . . . . . . . . . . 12
1.7. PRINCIPIO DE LAS CASILLAS. . . . . . . . . . . . . . . . . . . . . . . . . . . . . 13
1.8. PROBLEMAS . . . . . . . . . . . . . . . . . . . . . . . . . . . . . . . . . . . . . . . 14

2. Combinaciones y el Número Combinatorio 18


2.1. MODELO DE CONJUNTOS . . . . . . . . . . . . . . . . . . . . . . . . . . . . . . 18
2.2. MODELO DE CAMINOS . . . . . . . . . . . . . . . . . . . . . . . . . . . . . . . . 20
2.3. CADENAS DE CEROS Y UNOS . . . . . . . . . . . . . . . . . . . . . . . . . . . . 23
2.4. TRIÁNGULO DE PASCAL . . . . . . . . . . . . . . . . . . . . . . . . . . . . . . . 24
2.5. BINOMIO DE NEWTON . . . . . . . . . . . . . . . . . . . . . . . . . . . . . . . . 25
2.6. PROBLEMAS . . . . . . . . . . . . . . . . . . . . . . . . . . . . . . . . . . . . . . . 27

3. Permutaciones y Arreglos 33
3.1. PERMUTACIONES . . . . . . . . . . . . . . . . . . . . . . . . . . . . . . . . . . . 33
3.2. PERMUTACIONES CIRCULARES . . . . . . . . . . . . . . . . . . . . . . . . . . 33
3.3. PERMUTACIONES CON REPETICIÓN . . . . . . . . . . . . . . . . . . . . . . . 34
3.4. ARREGLOS . . . . . . . . . . . . . . . . . . . . . . . . . . . . . . . . . . . . . . . . 36
3.5. PROBLEMAS . . . . . . . . . . . . . . . . . . . . . . . . . . . . . . . . . . . . . . . 36

4. Extensiones del Número Combinatorio 39


4.1. SEPARADORES . . . . . . . . . . . . . . . . . . . . . . . . . . . . . . . . . . . . . 39
4.2. MULTICOMBINATORIO . . . . . . . . . . . . . . . . . . . . . . . . . . . . . . . . 40
4.3. PROBLEMAS . . . . . . . . . . . . . . . . . . . . . . . . . . . . . . . . . . . . . . . 41

5. Principios de Conteo 2 44
5.1. PRINCIPIO DE INCLUSIÓN - EXCLUSIÓN . . . . . . . . . . . . . . . . . . . . . 44
5.2. DESÓRDENES . . . . . . . . . . . . . . . . . . . . . . . . . . . . . . . . . . . . . . 44
5.3. RECURRENCIA . . . . . . . . . . . . . . . . . . . . . . . . . . . . . . . . . . . . . 45
5.4. PROBLEMAS . . . . . . . . . . . . . . . . . . . . . . . . . . . . . . . . . . . . . . . 47

2
ÍNDICE ÍNDICE

Introducción
Una manera muy sencilla de definir a la Combinatoria es entenderla como aquella área de la ma-
temática que trata el problema de contar.1 Esta actividad tan natural y a la que nos aproximamos
desde edades muy tempranas, tiene dos caracterı́sticas centrales:

1. Para contar, hay que considerar todas las posibilidades.

2. Y además, hay que asegurarse que cada objeto de conteo se cuenta exactamente una vez,
es decir, hay que evitar contar dos o más veces a un mismo objeto.

Es decir, cada objeto se cuenta al menos una vez y a lo sumo una vez. Sı́, esto es evidente,
pero pronto podrá darse cuenta que esto puede resultar muy complicado de percibir.

Para contar hay muchas formas, y la más elemental es hacer un conteo exhaustivo, es decir, uno
por uno elaborando un listado completo de los objetos, o lo que es lo mismo, un censo. Ahora bien,
si lo que nos proponemos contar es la cantidad de números de un millón de cifras que comienzan
con el dı́gito 4, obviamente, no haremos un censo, nunca terminarı́amos; allı́ entra en juego la com-
binatoria, la combinatoria proporciona métodos y técnicas para resolver este y otros problemas
en el que el conteo exhaustivo no funciona. Por tal motivo, algunos consideran a la combinatoria
como el arte de “contar sin contar”.2

Para llevar a cabo los conteos, se necesitarán de ciertos teoremas, que por lo evidente de su
veracidad, se les llama principios. A continuación se hace una breve aproximación a los principios
más importantes en combinatoria, y a lo largo del texto se tendrá la oportunidad de ir afinando
la concepción de los principios ası́ como agudizando la destreza de sus usos.

1
La Combinatoria es por supuesto mucho más extensa que esto, pero para nuestros fines, esta definición es la
mejor.
2
Por lo general, la matemática resuelve los problemas que le incumbe con métodos ingeniosos que desafı́an al
sentido común.

3
1 Principios de Conteo

1. Principios de Conteo
En este apartado desarrollaremos algunos principios básicos de conteo y enumeración. Cuando nos
interesamos en determinar el número de elementos en un conjunto dado, estamos en un caso de
conteo; mientras que cuando nos interesa listar los elementos estamos en el caso de enumeración.
Ambos problemas son importantes; hay situaciones en las que nos interesa no sólo saber cuántos
elementos hay en un conjunto dado, si no además saber cuáles son tales elementos, de aquı́ que
con frecuencia los métodos de conteo y enumeración son inseparables. El propósito de este apar-
tado es el de desarrollar algunas técnicas fundamentales de conteo en los que la enumeración no
aparece de manera explı́cita; se basan fundamentalmente en algunos principios cuya simplicidad
con frecuencia impide valorar su potencia; desarrollar la habilidad de aplicar correctamente tales
principios requiere alguna práctica por lo que se proponen ejercicios diversos para la aplicación de
los mismos en diferentes contextos.

1.1. PRINCIPIO DE LA SUMA


Un concepto fundamental cuando se trata de contar, es el de cardinal de un conjunto. En el caso
de los conjuntos finitos, el cardinal de un conjunto A es simplemente el número de elementos que
posee y se acostumbra denotarlo por |A| o bien por Card(A).

La técnica más elemental para contar los elementos de un conjunto es la de separar sus elementos
en clases disjuntas, de forma tal que su reunión incluya todos los elementos del conjunto. En otras
palabras se requiere que cada elemento del conjunto debe pertenecer a una sola de las clases y que
todo elemento del conjunto pertenece a una de las clases en las que se separa el conjunto.

Definición 1.1. Un conjunto finito A ha sido separado en n clases disjuntas A1 , A2 , A3 , . . . , An ,


si se satisface simultáneamente:

i. Ai ∩ Aj = ∅ para todo i 6= j.

ii. ni=1 Ai = A.
S

También decimos que el conjunto A ha sido particionado y las clases disjuntas (o exlcuyentes) se
denominan los elementos de la partición.

Teorema 1.1. Principio de la Suma: Si el conjunto A es posible separarlo en clases A1 , A2 , . . . , An ,


el total de elementos de A, es igual a la suma de los cardinales de cada una de las clases. Es decir

|A1 | + |A2 | + · · · + |An | = |A|

El principio de la suma suele ser enunciado también de la siguiente forma: si un suceso A puede
ocurrir de n maneras, un suceso B puede ocurrir de m maneras y ambos sucesos no pueden ocurrir
simultáneamente, entonces el suceso A o B puede ocurrir de (n + m) formas. Por supuesto tiene
su versión cuando hay más de dos sucesos, con la condición obvia que dos cualesquiera de ellos no
pueden ocurrir simultáneamente.

4
1.2 PRINCIPIO DE LA MULTIPLICACIÓN 1 Principios de Conteo

1.2. PRINCIPIO DE LA MULTIPLICACIÓN


Cuando se trata de contar parejas (x, y) conociendo el número de opciones de cada una de las
componentes del par, se utiliza el conocido como principio de la multiplicación que afirma
que el total de posibles pares que se pueden formar es el producto del número de alternativas que
se dispone para la primera componente por el número de alternativas para la segunda componente.

Por ejemplo si se lanza al aire un dado dos veces y anotamos los posibles resultados, estos los
podemos registrar mediante un par (x, y) registrando en la primera componente el resultado de la
primera tirada y en la segunda componente el resultado de la segunda tirada. Siendo que en cada
tirada hay seis posibles resultados: 1, 2, 3, 4, 5, 6, el total de posibles pares es 36. En efecto para
cada uno de los posibles resultados para la primera tirada tenemos 6 posibles resultados para la
segunda tirada y siendo que hay seis posibles resultados en la primera tirada, el total de resultados
será de 36.

De manera completamente análoga suponga que deseamos determinar el total de secuencias de


tres letras, es decir ternas (x, y, z) que se pueden formar con las letras a, b, c, d, e, f , de forma tal
que no se permite la repetición de letras. Habiendo 6 opciones de letras por colocar en la primera
posición, para la segunda posición sólo tendremos 5 opciones puesto que no se acepta la repetición
de letras en la terna; restricción que nos deja en la tercera posición sólo con 4 letras como posibles
opciones. Ası́ el total de ternas con la restricción planteada será de 6 · 5 · 4 = 120.

Teorema 1.2. Principio del Producto: Si A1 , A2 , A3 , . . . , Ak es una sucesión de conjuntos con car-
dinales n1 , n2 , n3 , . . . , nk respectivamente, entonces el conjunto que se obtiene haciendo el producto
cartesiano de tales conjuntos tiene por cardinal el producto de los cardinales de los conjuntos dados.
Es decir
|A1 × A2 × · · · × Ak | = n1 n2 · · · nk
En particular, cuando se trata del mismo conjunto A de cardinalidad n en cada uno de los factores,
se obtiene
k veces
z }| {
| A × A × · · · × A | = nk

En otra versión del principio de la multiplicación, éste se presenta en la forma siguiente: si un


conjunto puede ser particionado en k clases y cada una de las clases puede ser separada en t tipos
de elementos, el total de tipos de elementos es kt.

Otra versión muy frecuente del principio de la multiplicación se presenta de la forma siguiente:
si hay n1 alternativas de seleccionar un primer objeto y para seleccionar un segundo objeto se
dispone de n2 formas, la selección del par ordenado de objetos puede ser realizada en n1 n2 formas.

Analice el siguiente problema: Si desde Santa Ana a San Salvador se puede viajar por 3 rutas
distintas de autobus, y de San Salvador a San Miguel se puede viajar por 5 rutas distintas de
autobus, ¿de cuántas formas distintas se puede viajar en autobus desde Santa Ana hasta San
Miguel pasando por San Salvador? ¡Inténtelo!

5
1.3 PRINCIPIO DE BIYECCIÓN 1 Principios de Conteo

1.3. PRINCIPIO DE BIYECCIÓN


Consideremos el problema siguiente: En un campeonato de fútbol se enfrentan n equipos. En cada
ronda los equipos perdedores son eliminados. Si en una ronda el número de equipos aún partici-
pando es impar, uno de los equipos, elegido mediante sorteo, descansa y pasa a la ronda siguiente.
¿Cuántos juegos se realizan durante el campeonato?

Este problema tiene por supuesto muchas maneras de ser abordado, pero en este caso vamos a
destacar la forma que se apoya en el denominado principio de correspondencia.

Definición 1.2. Dos conjuntos A y B son tales que a cada elemento de A se le puede asociar uno
y sólo un elemento de B y viceversa (a cada elemento de B se le puede asociar uno y sólo uno de
los elementos de A), entonces decimos que A y B pueden ponerse en correspondencia biyectiva.
En este caso se dice también que los conjuntos A y B son coordinables y escribimos en forma
simbólica A ∼= B.3
Teorema 1.3. Principio de Correspondencia: Si A y B son coordinables entonces tienen el mismo
número de elementos o cardinal.

Veamos cómo se aplica el principio anterior al problema propuesto: Como en cada partido hay un
perdedor y sólo uno y para cada perdedor hay uno y un único partido, para contar el número de
partidos nos basta contar el número de perdedores y siendo que al final del campeonato sólo queda
uno de los equipos, el número de perdedores es n − 1, que debe por supuesto ser el número de
partidos jugados en el campeonato. Ası́, este número debe ser n − 1.

El principio anterior es importante cuando la tarea de contar los elementos de un conjunto A


resulta más difı́cil que la de contar en un conjunto B que puede ponerse en correspondencia biyec-
tiva con A. Se cuenta entonces en el conjunto B y automáticamente se ha contado en el conjunto A.

He aquı́ otro ejemplo en donde se aplica el principio de correspondencia biyectiva: Queremos


determinar el total de formas que tenemos de seleccionar 9 elementos en un conjunto que posee 10
elementos. Siendo que hay una correspondencia biyectiva entre los conjuntos de 9 elementos con
los conjuntos de 1 elemento, es suficiente para resolver el problema contar el número de selecciones
de 1 elemento, que son por supuesto 10 formas.

FFF
Analicemos ahora algunas ejemplos un poco más complicados que utilizan estos tres principios
fundamentales; con frecuencia, en un mismo problema se utiliza más de uno.

EJEMPLO 1.1
Un profesor tiene 35 estudiantes en el curso de Álgebra y 38 estudiantes en el curso de Geometrı́a.
¿Cuántos estudiantes tiene en total?

3
Un conjunto A se dice que es finito si es vacı́o o si es coordinable con un conjunto de la forma {1, 2, 3, . . . , n} y
en este último caso se dice que el cardinal de A es n, o bien que el número de elementos que posee es n. Cuando A
es vacı́o decimos que el conjunto tiene cero elementos.

6
1.3 PRINCIPIO DE BIYECCIÓN 1 Principios de Conteo

La respuesta 73 estudiantes sólo serı́a válida en el caso de que ningún estudiante reciba los dos
cursos impartidos por el profesor. Si hay estudiantes que reciben ambos cursos, el conteo exigirı́a
que se separe el total de estudiantes en las tres clases disjuntas: los que reciben sólo el curso de
Álgebra, los que reciben sólo el curso de Geometrı́a y los que reciben ambos cursos. Suponga
por ejemplo que hay 10 estudiantes que reciben ambos cursos, entonces los conjuntos no serı́an
disjuntos; si descontamos los estudiantes comunes del curso de Álgebra y Geometrı́a, tendrı́amos
25 que reciben sólo el curso de Álgebra y 28 estudiantes que reciben sólo el curso de Geometrı́a.
Ası́ el total es la suma del número de elementos que poseen los tres conjuntos disjuntos, es decir:
25 + 28 + 10 = 63.

EJEMPLO 1.2
Se tienen 6 libros distintos de Álgebra, 5 libros distintos de Geometrı́a, y 4 libros distintos de
Trigonometrı́a. ¿De cuántas formas es posible seleccionar un par no ordenado de libros que no sean
de la misma asignatura?

Siendo que los libros seleccionados deben ser de asignaturas diferentes, las posibilidades de combi-
nar son: Un libro de Álgebra y uno de Geometrı́a; uno de Álgebra y uno de Trigonometrı́a y por
último uno de Geometrı́a y uno de Trigonometrı́a; con ello hemos separado los pares posibles en
casos disjuntos. Aplicando en cada uno de los casos el principio de la multiplicación tenemos que
las alternativas para cada caso son: 6 · 5, 6 · 4 y 5 · 4, respectivamente. Ası́, por el principio de la
suma, el total de alternativas para hacer la selección del par de libros es: 30 + 24 + 20 = 74.

EJEMPLO 1.3
Se consideran las cadenas de ceros y unos de longitud 5, es decir secuencias con cinco caracteres
entre los dı́gitos cero y uno. Por ejemplo la cadena 00010 es de longitud cinco. ¿Cuál es el total de
cadenas de ceros y unos de longitud 5?

Observe que si P = {0, 1} , cada una de las cadenas puede ser identificada mediante el quinteto
(x1 , x2 , x3 , x4 , x5 ) del producto cartesiano P × P × P × P × P = P 5 , recı́procamente, todo elemento
del producto cartesiano P 5 está asociado a una cadena de ceros y unos de longitud cinco. En
consecuencia, dada esta correspondecia biyectiva, el total de cadenas será |P 5 | = 25 = 32.

EJEMPLO 1.4
¿Cuál es el número de subconjuntos que posee un conjunto A de cardinal n?

Como en el ejemplo anterior, nos apoyaremos en la correspondencia biyectiva entre los subconjuntos
de A con las cadenas de longitud n. Supongamos ordenados los elementos de A del primero al
n-ésimo elemento y adoptemos la convención siguiente: a cada subconjunto de A asociamos la
cadena de longitud n, cuya componente en la posición k-ésima es 0 si el elemento k-ésimo de A no
pertenece al subconjunto, o bien 1 si el elemento pertenece al subconjunto. Por ejemplo la cadena
(0, 0, 0, 0, . . . , 0) está asociada al subconjunto vacı́o; el subconjunto con sólo el primer elemento de
A se asocia con la cadena (1, 0, 0, 0, . . . , 0) que posee sólo ceros salvo en la primera componente.
Con esta convención se logra en efecto la correspondencia biyectiva y podemos, en vez de contar
subconjuntos de A, contar las cadenas de ceros y unos de longitud n, que como extensión del
ejemplo anterior será |P n | = 2n (utilizamos el mismo conjunto P del ejemplo anterior).

7
1.3 PRINCIPIO DE BIYECCIÓN 1 Principios de Conteo

EJEMPLO 1.5
Usando las letras a, b, c, d, e, ¿de cuántas formas es posible formar una secuencia ordenada de tres
letras en cada uno de los siguientes casos?
a) Si la repetición de letras está permitida.

b) Si no se permite la repetición de letras.

c) Sin repetición de letras y que la secuencia tenga a la letra e.

d) Con repetición de letras conteniendo e.

a) En el primero de los casos como la repetición está permitida, el problema es equivalente a


determinar el número de ternas del conjunto A3 , en donde A = {a, b, c, d, e}, que es 53 = 125.

b) En el segundo caso no se permite la repetición, se debe tomar en cuenta que para formar las
ternas solicitadas, para la primera componente de la terna disponemos de todos los elementos
del conjunto A; sin embargo para la segunda componente, disponemos de un conjunto con
cuatro elementos puesto que debemos excluir el elemento colocado en la primera componente;
de manera similar para la tercera componente sólo dispondremos de un conjunto con tres
elementos habiendo excluido los pertenecientes a la primera y segunda componente de la terna.
Ası́, el total de ternas con la condición solicitada es: 5 · 4 · 3 = 60.

c) Para el tercer caso, en el que la letra e está obligada a pertenecer a la secuencia, podemos
dividir en casos de acuerdo a la posición que ocupe la letra e en la terna, que son obviamente
casos disjuntos. Ası́, si ocupa la primera posición las ternas son de la forma (e, x2 , x3 ) y para la
segunda componente se dispone de cuatro elementos en tanto que para la tercera componente
se dispone nada más que de tres elementos, por lo que en este caso tendremos 4 · 3 = 12
posibilidades. Los otros casos a considerar son las ternas de la forma (x1 , e, x3 ) y las ternas
de la forma (x1 , x2 , e) que son igualmente 12 posibilidades en cada uno, por lo que en total
tendremos 36 secuencias con la condición dada.

d) Para el último caso en el cual se permite repetición pero la secuencia contiene obligatoriamente
a la e, haremos la separación de ternas en las clases siguientes: (e, x2 , x3 ), (x1 , e, x3 ) y (x1 , x2 , e),
tomando en cuenta que en la primera clase tendremos para x2 y x3 cinco posibilidades, es decir
tendremos 25 ternas en esta primera clase; en la segunda clase habrá que excluir x1 = e,
puesto que las ternas con e en la primera componente ya fueron contadas en las ternas de la
primera clase, ası́, en esta segunda clase de ternas tendremos 4 posibilidades para la primera
componente y 5 para la tercera componente, es decir tendremos 20 ternas en esta clase; en la
tercera clase debemos excluir los casos x1 = e y x2 = e puesto que las ternas de este tipo ya
fueron consideradas en las dos clases anteriores y en consecuencia tendremos 4 posibilidades
para x1 y 4 posibilidades para x2 , resultando 16 ternas de esta clase. En total tendremos
25 + 20 + 16 = 61 ternas. ¡Recuerde que contar significa tomar en cuenta todos los elementos
del conjunto pero cada elemento tomado en cuenta sólo una vez!

EJEMPLO 1.6
Considere una cuadrı́cula 8 × 8. Determine el número de cuadrados formados por vértices de la

8
1.4 INCLUSIÓN-EXCLUSIÓN 1 Principios de Conteo

cuadrı́cula cuyos lados son paralelos a los segmentos de la cuadrı́cula.

Este es un tı́pico problema en el que se evidencia la importancia de contar con método, de lo


contrario la tarea se vuelve complicada. Tal como lo demanda el principio de la suma, lo mejor es
separlos en clases disjuntas. Ası́, contaremos por separado los cuadrados de lado 1, los cuadrados
de lado 2 y ası́ sucesivamente hasta los cuadrados de lado 8.
Los cuadrados de lado 1 son 8 por cada fila y siendo que tenemos 8 filas, el total de cuadrados
de lado 1 es 8 · 8 = 64. Para los cuadrados de lado 2 observe que para formarlos se requiere
un par de filas consecutivas y en ellas los cuadrados de lado 2 que pueden formarse son 7; si se
toma en cuenta que se pueden formar siete pares de filas consecutivas se obtiene que el total de
cuadrados de lado 2 es 7 · 7 = 49. De forma completamente análoga para formar cuadrados de
lado 3 requerimos disponer de tres filas consecutivas y en ellas es posible construir 6 cuadrados de
lado 3; como se pueden formar seis trı́os de filas consecutivas, el número de cuadrados de lado 3
será de 6 · 6 = 36 cuadrados. Después de considerar los casos anteriores los siguientes casos son
fáciles de deducir: el total de cuadrados de lado 4 será 5 · 5, el de cuadrados de lado 5, será 4 · 4,
los de lado 6, serán 3 · 3, los de lado 7 serán 2 · 2 y los de lado 8, 1 · 1. Ası́ el total de cuadrados
buscado será 1 · 1 + 2 · 2 + 3 · 3 + 4 · 4 + 5 · 5 + 6 · 6 + 7 · 7 + 8 · 8 = 204 cuadrados.

EJEMPLO 1.7
Determine el número de diagonales que posee un decágono.

Por supuesto que dibujar las diagonales y después contarlas es una de las opciones para resolver el
ejercicio, pero obviamente es una solución poco práctica y difı́cil de generalizar (y también difı́cil
porque el dibujo puede complicarse demasiado). Vamos a utilizar la separación de las diagonales
en casos disjuntos para en seguida aplicar el principio de la suma. Recuerde que en un polı́gono
se denomina diagonal al segmento que une dos vértices no consecutivos del polı́gono. Para separar
en clases disjuntas vamos a considerar las diagonales que “salen” de cada uno de los vértices.
Dado que en un decágono cada vértice tiene 7 vértices no consecutivos son siete diagonales que
salen de cada vértice; como en un decágono tenemos 10 vértices, el total de diagonales será de 35
tomando en cuenta que cada diagonal ha sido contada dos veces, una vez por cada extremo de la
diagonal.

1.4. INCLUSIÓN-EXCLUSIÓN
Clasificar, como ya se habrá observado, es un método importante cuando se trata de contar obje-
tos. Supone sin embargo que somos capaces de particionar el conjunto dado en clases disjuntas y
esto no siempre es sencillo. Es necesario desarrollar otros métodos.

Una estrategia que en ciertos casos ofrece alguna ventaja, es la de usar el criterio de contar en
el complemento, en vez de contar directamente en el conjunto dado. Los dos ejemplos siguientes
ilustran este caso.

EJEMPLO 1.8
De las cadenas binarias de ceros y unos, de longitud 10, ¿cuántas tienen la propiedad de poseer
por lo menos un 0?

9
1.5 RECURRENCIA. 1 Principios de Conteo

EJEMPLO 1.9
Cinco niños solicitan, en una sorbeterı́a que dispone de sorbetes de 8 diferentes sabores, un sorbete
cada uno. De todas las posibles solicitudes que pueden hacer, ¿en cuántas de ellas los sabores
elegidos por los niños, coinciden en cuando menos dos de ellos?
En la solución de los dos problemas anteriores aún cuando es posible clasificar en conjuntos dis-
juntos de manera directa, el número de clases que resulta es demasiado alto y es preferible contar
por su complemento.

Hay ocasiones sin embargo en las que la clasificación en el conjunto o en su complemento ofrece el
mismo nivel de dificultad y en este caso el principio de inclusión-exclusión resulta ser un valioso
recurso de conteo.

En su versión más elemental, el principio de inclusión- exclusión nos dice que el número de ele-
mentos en la reunión de dos conjuntos A y B, se calcula haciendo la suma de los números de
elementos A mas los de B y se resta el número de elementos comunes a ambos conjuntos, es decir
los elementos de la intersección de ambos conjuntos. La interpretación es obvia: la resta del número
de elementos de la intersección de los conjuntos evita que éstos se cuenten dos veces por cuanto
están incorporados como elementos del conjunto A y como elementos del conjunto B. En expresión
conjuntista, tenemos:
|A ∪ B| = |A| + |B| − |A ∩ B|
En el caso de la reunión de tres conjuntos A, B, C, los ajustes que debemos realizar para calcular
el número de elementos de la reunión de los tres conjuntos son menos obvios. En este caso, si
sumamos el número de elementos de A, B y C, observamos que los elementos comunes a dos
de tales conjuntos han sido contados dos veces, mientras que los elementos comunes a los tres
conjuntos han sido contados tres veces. Procedemos a corregir, restando los elementos comunes a
dos de los conjuntos dados; en este caso tenemos tres posibilidades A con B; A con C; B con C.
Con esta corrección los elementos estrictamente pertenecientes a dos de los conjuntos dados quedan
contados sólo una vez; sin embargo, los elementos comunes a los tres conjuntos, que anteriormente
habı́an sido contados tres veces, ahora han sido descontados tres veces, lo que hace que en suma
no hayan sido contados. Hacemos ahora la corrección final sumando el número de elementos que
son comunes a los tres conjuntos. Hemos obtenido en consecuencia el resultado siguiente:

|A ∪ B ∪ C| = |A| + |B| + |C| − (|A ∩ B| + |B ∩ C| + |C ∩ A|) + |A ∩ B ∩ C|

1.5. RECURRENCIA.
La recursión es un poderoso método de análisis que consiste en suponer que se conocen los valores
de una función de conteo para todos los valores previos a n, e intentar determinar, a partir de tal
información, el valor de tal función para el valor de n.

Cuando se trata de contar objetos en función de una o más variables, la condición ideal es la de
lograr una fórmula explı́cita que relacione tales variables. En ciertas ocasiones, sin embargo, tal
condición resulta difı́cil de alcanzar y con frecuencia es suficiente obtener las suficientes relaciones
que nos permitan generar un algoritmo de cálculo para la resolución de nuestro problema en el caso
n-ésimo, a partir de la información acerca de los casos previos, es decir buscar una recurrencia;

10
1.5 RECURRENCIA. 1 Principios de Conteo

en otras ocasiones incluso es preferible y más sencillo buscar tales relaciones. He aquı́ algunos de
ejemplos de cómo esta técnica permite resolver algunos problemas:

EJEMPLO 1.10
Determinar el número de subconjuntos de un conjunto finito dado.

Podemos determinar el número de subconjuntos de un conjunto finito dado a partir de una re-
currencia. En efecto, el número de subconjuntos de un conjunto unitario es 2, y si denotamos el
número de subconjuntos de un conjunto con n elementos como Sn , el número de subconjuntos en
el caso de un conjunto con (n + 1) elementos puede ser calculado de la siguiente manera: consi-
deremos el último elemento incorporado, entonces sólo hay dos posibilidades de subconjuntos, los
que no incluyen el elemento último que son en total Sn , y los que sı́ lo incluyen, que se obtienen
introduciendo éste en cada uno de los subconjuntos de los n primeros elementos, que de nuevo son
Sn . Hemos demostrado ası́ que:

Sn+1 = 2Sn
Esta relación junto con la condición inicial S1 = 2, permite calcular todos los valores de Sn , de
manera recurrente, para cualquier número natural n

Sn = 2Sn−1
= 2 (2Sn−2 ) = 22 Sn−2
= 22 (2Sn−3 ) = 23 Sn−3
..
.
= 2n−1 S1 = 2n

EJEMPLO 1.11
Determinar el número de diagonales que posee un polı́gono convexo de n lados.

Aún cuando no podamos determinar tal número, podemos intentar establecer una relación entre
el número de diagonales de un polı́gono de n lados con las de un polı́gono de (n + 1) lados. Sea Dn
el número de diagonales de un polı́gono de n lados. De lo que se trata es de establecer una relación
entre Dn y Dn+1 . Si agregamos un vértice a un polı́gono de n lados, éste lo podemos imaginar
ubicado entre dos de los n vértices antiguos. Este vértice puede ser conectado con n − 2 vértices
antiguos para formar diagonales; ası́ se formarán n − 2 nuevas diagonales; sin embargo, un lado
en el antiguo polı́gono (justamente el lado que unı́a los dos vértices vecinos al nuevo vértice) se ha
transformado en diagonal; en otras palabras el nuevo vértice ha generado (n−1) nuevas diagonales.
Concluimos entonces que debe cumplirse la relación de recurrencia:

Dn+1 = Dn + (n − 1)
Por otra parte, observe que D3 = 0. La información anterior es suficiente para calcular el número de
diagonales de cualquier polı́gono, simplemente aplicando de manera iterativa la relación anterior.
¡Inténtelo!

11
1.6 INDUCCIÓN MATEMÁTICA. 1 Principios de Conteo

1.6. INDUCCIÓN MATEMÁTICA.


La inducción matemática es una forma de demostración muy útil para problemas de combina-
toria, ası́ como de otras áreas. La inducción tiene un vı́nculo muy estrecho con la relaciones de
recurrencia, de alguna manera, la inducción es un proceso en la dirección contraria a la recurrencia.

La idea intuitiva de la inducción es la de fichas de dominó, que puestas de pie una después de otra
en una fila y todas muy cercanas, caen todas si se asegura que cae una de ellas, porque la primera
que cae bota a la segunda, la segunda a la tercera, y se sigue ası́ indefinidamente.

De igual forma, un problema que se resuelve utilizando inducción requiere una hipótesis que:
Se verifique su veracidad para algún caso particular.
La veracidad de la hipótesis para un caso k implique la veracidad de la hipótesis para un
caso k + 1.
Si esto es ası́, entonces la hipótesis será cierta para todos los casos mayores al caso inicial, por-
que, dado que el primer caso es cierto, éste obliga a que el segundo lo sea, y como el segundo es
cierto, este obliga a que el tercero lo sea, y ası́ sucesivamente. Veamos la inducción en un la práctica:

EJEMPLO 1.12
¿Cuántos subconjuntos tiene un conjunto de cardinalidad n?

Este problema ya lo resolvimos anteriormente utilizando recurrencia, ahora lo haremos con induc-
ción y vea que ambas aproximaciones son muy parecidas.

Como todo problema que exige un resultado general, es fundamental el análisis de casos par-
ticulares; por ejemplo, podemos comenzar contando cuántos subconjuntos tiene un conjunto de
n = 1, 2, 3 elementos, haciendo listados de forma exhaustiva.
Caso n = 1: Considere el conjunto A = {x}, los subconjuntos de A son ∅ y {x}, es decir, S1 = 2
(utilizamos la misma notación que antes, Sn representa la cantidad de subconjuntos de un conjunto
de cardinalidad n).
Caso n = 2: Considere ahora el conjunto B = {x, y}, los subconjuntos de B son ∅, {x}, {y} y
{x, y}, por lo que S2 = 4.
Caso n = 3: Sea C = {x, y, z}, los subconjuntos de C son ∅, {x}, {y}, {z}, {x, y}, {y, z}, {z, x},
{x, y, z}, por lo que S3 = 8.
Ahora se ve un patrón, pero para estar seguros, lo mejor es hacer unos casos más; la siguiente
tabla muestra un resumen lo lo obtenido:
n 1 2 3 4 5
Sn 2 4 8 16 32
Parece entonces que Sn es una potencia de 2, y si somos más cuidadosos, observarı́amos que Sn
es 2n para n = 1, 2, 3, 4, 5; bueno, esto es una conjetura obtenida a partir de casos particulares,
todavı́a el problema no está resuelto porque no se ha garantizado que Sn = 2n para todo natural
n, pero eso es lo que haremos enseguida.

12
1.7 PRINCIPIO DE LAS CASILLAS. 1 Principios de Conteo

Nuestra conjetura ahora le daremos el valor de Hipótesis Inductiva, vamos a suponer que Sn = 2n
para algún natural n, entonces ¿qué sucede si agregamos un elemento?, los subconjuntos del nuevo
conjunto son: 1) Todos los subconjuntos del conjunto original de n elementos, y 2) los subconjuntos
que se forman agregando el nuevo elemento a los subconjuntos anteriores, es decir (y tal como lo
hicimos en recurrencia), Sn+1 = 2Sn , pero por nuestra asumpción resulta que Sn+1 = 2 (2n ) = 2n+1 .
Entonces ¿qué se demostró? bien, se demostró que si Sn es una potencia de 2 entonces Sn+1 es
la siguiente potencia de 2, y dado que S1 = 21 es una potencia de 2, entonces S2 , S3 , . . . , Sn , . . .
recorren todas las restantes potencias de 2, y el resultado Sn = 2n es válido ahora para todo natural
n.
Es importante mencionar dos cosas:

Es de vital importancia que nuestra hipótesis inductiva se garantice para algún caso particu-
lar, de nada sirve que el caso n obligue al caso n + 1 si no se tiene garantizado que habrá un
caso inicial, que empuje al segundo, y que este empuje al tercero, etc. Esto serı́a equivalente
a que las fichas de dominó se pusieran muy cerca y se hacen todas las preparaciones posibles
que garanticen que si una cae caerá la siguiente, pero NADIE se toma la molestia de empujar
alguna, en tal caso, pues nunca caerá ficha alguna.

Por otra parte, se puede suponer que la hipótesis inductiva es cierta para un n, o bien, que
es cierta para todos los k menores o iguales a n, de cualquier forma, lo importante es que
esta suposición implique la veracidad de la hipótesis inductiva para el caso n + 1.

1.7. PRINCIPIO DE LAS CASILLAS.


El principio de las casillas de Dirichlet, también conocido como el principio del palomar o principio
de los cajones, es un principio fácil de aceptar en su versión más elemental. Su aplicación invade
ámbitos muy diversos y con frecuencia su utilización permite resolver problemas que distan mucho
de ser triviales.

La idea central del principio de casillas es la siguiente: si se tienen n + 1 bolas que serán colocadas
en n contenedores (las casillas), entonces forzosamente hay un contenedor que tendrá dos o más
bolas, ¡lógico! ¿cierto? A continuación se muestran otras versiones, pero que en escencia son lo
mismo

Teorema 1.4. Principio de Casillas:


Versión 1: Si más de n bolitas se encuentran distribuidas en n cajas entonces hay una caja que
contiene al menos 2 bolitas.
Versión 2: Si más de mn bolitas se encuentran distribuidas en n cajas entonces hay una caja que
contiene al menos m + 1 bolitas.
Versión 3: Si la suma de n cantidades es mayor que S entones una de las cantidades es mayor que
S
.
n
Versión 4: Un segmento I de longitud t contiene varios segmentos cuyas longitudes suman más
que t, entonces al menos dos de los segmentos contenidos en I se solapan.
Versión 5: Un segmento I de longitud t contiene varios segmentos cuyas longitudes suman más
que kt, donde k es un entero positivo, entonces hay un punto del segmento I que está contenido

13
1.8 PROBLEMAS 1 Principios de Conteo

en k + 1 de los segmentos contenidos en I.


También hay versiones análogas a las últimas dos para figuras planas y áreas. Versión 6: Sean
q1 , q2 , . . . , qn enteros positivos. Si q1 + q2 + · · · + qn − n + 1 objetos son colocados en n cajas,
llamadas C1 , C2 , . . . , Cn , entonces al menos una caja Ci tiene qi objetos o más.

EJEMPLO 1.13
De un conjunto de 12 números enteros de dos dı́gitos, siempre podemos seleccionar dos de ellos
cuya diferencia tiene la forma aa.4

Obsérvese primero que los números de la forma aa son todos los múltiplos de 11 de dos cifras. Ahora
bien, si hacemos la división de los números dados por 11, los posibles residuos son 0, 1, 2, . . . , 10,
y como se escogieron 12 números, hay por lo menos una pareja de números que tienen el mismo
residuo, entonces su diferencia será divisible por 11 y deberá ser de la forma aa.

1.8. PROBLEMAS
1. Cinco jueces de un deporte determinado disponen de una cartulina en la que por un lado
hay un 1 y por el otro un 0. ¿Cuántas configuraciones distintas pueden generar?

2. Un marino tiene 4 banderas distintas para hacer señales. ¿Cuántas señales diferentes puede
hacer si coloca 3 banderas en un mástil una sobre otra?

3. El ascensor de un edificio de 5 plantas se pone en marcha con 3 pasajeros. ¿De cuantos modos
distintos se pueden distribuir los pasajeros entre los pisos?

4. Se lanzan dos dados y se observa la suma de los puntos obtenidos. ¿En cuántos casos el
resultado es divisible por tres?

5. En cierto lenguaje de programación un nombre de una variable puede formarse con una letra
o bien una letra seguida de un un dı́gito. ¿Cuántas variables diferentes pueden formarse?
Asuma para este ejercicio 27 letras.

6. Se tiene una tira o banda formada por n rectángulos iguales formando una fila. Cada rectángu-
lo puede colorearse de blanco o negro.

a) ¿Cuántas configuraciones diferentes se pueden hacer?


b) ¿Cuántas formas distintas tenemos de colorear la tira, de modo que se obtenga un patrón
simétrico?
c) ¿Y si se desea con colores alternados?

7. ¿De cuantas maneras pueden colocarse un alfil blanco y uno negro en un tablero de ajedrez
de modo que se ataquen mutuamente?
4
La notación ab hace referencia a un número cuya cifra de las unidades es a y la cifra de las decenas es b, y la se
coloca la raya encima para evitar confundir este número con ab, que representa el producto de a por b. Esta claro
que esta notación puede utilizarse para representar números de tres, cuatro o más cifras.

14
1.8 PROBLEMAS 1 Principios de Conteo

8. ¿De cuántas formas es posible seleccionar dos cartas diferentes de una baraja de 52 cartas
de forma tal que la primera carta sea un as y la segunda no sea una reina? ¿y si se demanda
es que la primera carta sea de espadas y la segunda no sea reina?

9. ¿Cuántos números enteros hay entre 1000 y 10000 con la condición de que sus dı́gitos sean
diferentes? ¿y si se permite la repetición de dı́gitos, pero no se permiten los dı́gitos 2 ó 4? ¿y
cuando los dı́gitos deben ser distintos y al menos uno de los dı́gitos 2 y 4 debe aparecer?

10. Con los dı́gitos del 0 al 9, ¿cuántas secuencias de longitud 5 pueden ser formadas de forma
tal que aparezcan exactamente dos de los diez dı́gitos?

11. De cuántas formas pueden ser colocadas en un tablero de ajedrez dos torres idénticas en una
fila común o en una columna común? ¿y si las torres no son idénticas?5

12. ¿Cuántos números de cuatro dı́gitos se pueden formar con los dı́gitos 1, 2, 3, 4, 5 (con posible
repetición) tales que sean divisibles por 4?

13. ¿Cuántos cuadrados de longitud entera pueden formarse en una cuadrı́cula de n × n (con los
lados formados por segmentos de la cuadrı́cula)?

14. ¿Cuántas diagonales se pueden trazar en un polı́gono convexo de n lados?

15. Se dice que en un polı́gono se ha realizado una triangulación, cuando el interior del polı́gono
se cubre con triángulos formados con vértices del polı́gono de forma tal que los triángulos no
tengan en común más que conjuntos de puntos de área cero. ¿Cuántos triángulos se requieren
en una triangulación en un polı́gono de n lados?

16. ¿Cuántas secuencias de tres letras diferentes pueden ser formadas haciendo uso de las letras
a, b, c, d, e, f en las cuales aparecen la letra e, o la letra f , o ambas e y f ?

17. Cuando se listan los números del 1 al 10000, ¿cuántas veces se hace uso del dı́gito 5? ¿y
cuántas veces aparece el “25”?

18. En un tablero de ajedrez, de cuántas formas se pueden colocar dos reinas de forma tal que
no estén ni en la misma fila, ni en la misma columna ni en la misma diagonal?6

19. ¿Cuántos números positivos pueden formarse como suma de los números 1, 3, 5, 10, 20 y 50?

20. Considere una cuadrı́cula de 5 × 5 ¿cuántos rectángulos podemos formar con los vertı́ces de
dicha cuadrı́cula cuyos lados sean paralelos a los segmentos de la cuadrı́cula?

21. Una persona desea invitar a uno o más de sus 10 amigos a su fiesta de cumpleaños. ¿de
cuántas formas podrı́a hacer tal invitación?

22. ¿Cuántos números de tres cifras distintas pueden formarse con los dı́gitos impares?

23. Un turista debe trasladarse de una ciudad a otra. Para hacerlo pude optar viajar por avión,
autobus o tren, y en cada uno de estos medios puede elegir viajar en 1a clase o en clase
turı́stica. ¿De cuántas maneras distintas puede realizar el viaje?
5
Cuando un par de torres de ajedrez comparten fila o columna, decimos que las torres se están “atacando”.
6
Las reinas no se están “atacando”.

15
1.8 PROBLEMAS 1 Principios de Conteo

24. En una fiesta se encuentran 10 hombres y 8 mujeres. ¿De cuántas maneras pueden integrarse
en parejas para bailar una determinada pieza?

25. Un menú turı́stico permite seleccionar una entrada de entre cuatro posibles, una comida
caliente de entre tres, y un postre de entre cinco. ¿De cuántas formas puede elegir su menú un
turista si desea que el pollo guisado y el flan no aparezcan en el mismo menú?

26. ¿Cuántos boletos capicuas de 5 cifras hay?

27. Seis matrimonios posan en fila para una fotografı́a. ¿De cuántas maneras pueden ubicarse si
los miembros de cada pareja deben aparecer juntos?

28. Se desea colocar 9 libros distintos en tres estantes en una biblioteca, 3 en cada estante. ¿De
cuántas pueden ubicarse?

29. Al ingresar a un colegio, 6 chicas debe optar entre asistir a las clases de inglés o a las de
francés, ¿de cuántas maneras pueden repartirse si Paula y Daniela deciden elegir la misma
materia y Cecilia no quiere compartir la elección de ellas?

30. Un sábado, cuando iban de compras, Juana y Teresa vieron a dos hombres alejarse en au-
tomóvil de la fachada de una joyerı́a, justo antes de que sonará una alarma contra robos.
Aunque todo ocurrió muy rápido, cuando fueron interrogadas las dos jóvenes, pudieron dar
a la polica la siguiente información acerca de la placa (que consta de 2 letras seguidas de 4
dı́gitos) del automóvil que huyó: Teresa estaba segura de que la segunda letra de la placa era
una O o una Q, y el el último dı́gito era un 3 ó un 8. Juana dijo que la primera letra de la
placa era una C o una G y que el primer dı́gito era definitivamente un 7. ¿Cuántas placas
diferentes tendrá que verificar la policı́a?

31. ¿Cuántos números naturales menores o iguales a un millón no tienen dos cifras consecutivas
iguales?

32. ¿De cuántas maneras se pueden colocar 8 torres en un tablero de ajedrez tal que no se
ataquen?

33. ¿De cuántas maneras se puede escoger un cuadrado negro y uno blanco de un tablero de
ajedrez tal que no pertenezcan a la misma fila ni a la misma columna?

34. Pruebe que el número máximo de alfiles (todos iguales) que se pueden colocar en un tablero
cuadrado de n × n casillas sin que haya dos que se ataquen es 2n − 2, y que el número de
estas configuraciones es 2n .

35. Entre 20000 y 70000, encuentre el número de enteros pares en los cuales no se repita ningún
dı́gito.

36. Una emisora de radio dispone de seis programas para cubrir las dos horas de mayor audiencia.
Dos de estos programas tienen una duración de una hora y los cuatro restantes de 30 minutos,
siendo todos los programas distintos. ¿De cuántas maneras puede la emisora cubrir las dos
horas de programación?

37. ¿Cuántos números del 1 al 100000 no son divisibles ni por cinco ni por siete?

16
1.8 PROBLEMAS 1 Principios de Conteo

38. Tenemos los números del 100000 al 999999. ¿Cuántos de ellos cumplen que tienen al menos
un dı́gito cero o al menos un dı́gito uno?

39. Dado el conjunto de dı́gitos {1, 3, 6, 7, 9}, determine el número de formas de formar números
de 4 dı́gitos tales que sean múltiplos de 3.

40. ¿Cuántos divisores positivos tiene el número 121 0?

41. Marı́a y Laura idean el siguiente juego: cada una lanza un dado, si en los dados sale el mismo
número, gana Laura; si la suma de ambos es 7, gana Marı́a; y en cualquier otro caso hay
empate. Calcule las probabilidades de gane de cada una y diga cuál tiene mayores chances
de éxito.

42. ¿Cuántos cubos diferentes, con sus caras numeradas de 1 al 6 pueden ser fabricados, si la
suma de los números que se encuentran sobre cada par de lados opuestos debe ser 7?

43. ¿De cuántas maneras puede colorearse una cuadricula de 8 × 8 con los colores blanco y negro,
de tal manera que no existan dos columnas pintadas iguales?

44. ¿De cuántas manereas se pueden colocar las 32 piezas del ajedrez en el tablero sin que los
reyes se estén amenazando?

45. Dado el conjunto A = {1, 2, . . . , 10}, ¿cuántos cuadrados se pueden formar con los puntos de
A2 ?
Nota: los cuadrados pueden ser oblicuos también.

46. Se dibujan n cuerdas no concurrentes en una circunferencia (n ≥ 2). Estas cuerdas se cortan
en m puntos al interior de la circunferencia, los cuales subdividen a las cuerdas en r segmentos.
Determine r en función de n y m.

Nota: En la figura anterior, se han dibujado n = 4 cuerdas, que se cortan en m = 4 puntos,


los cuales subdividen a las cuerdas en r = 12 segmentos.

47. Sea X = {1, 2, 3, . . . , 100} y sea S = {(a, b, c) tales que a, b, c ∈ X, a < c y a < c}. ¿Cuál es
el cardinal de S?

17
2 Combinaciones y el Número Combinatorio

2. Combinaciones y el Número Combinatorio


2.1. MODELO DE CONJUNTOS
Dado un conjunto de n elementos (n ∈ N), ¿cuántos subconjuntos hay de k elementos (0 < k ≤ n)?
A este número le llamaremos el combinatorio de n escoger k.

Definición 2.1. Número Combinatorio: Dado un conjunto de cardinalidad n, el número de sub-


conjuntos de éste que tienen cardinalidad k es el número combinatorio, denotado por

Cnk , (nk ) , o bien nCk

Esta claro que este número puede interpretarse como la cantidad de formas de escoger k elementos
de entre los n disponibles.7

A pesar que de momento no podemos calcular este número, sı́ podemos hacer un uso habilidoso
de él a partir de su definición y de las propiedades que se derivan a partir de ésta.

Es importante mencionar que Cn0 significa la cantidad de subconjuntos de cardinalidad cero que
tiene un subconjunto de n elementos, y como el único subconjunto que cumple eso es ∅ entonces
Cn0 = 1 para cualquier natural n; también es fácil argumentar que Cnn = 1 para todo natural n.

EJEMPLO 2.1
Suponga que se tiene el conjunto A = {a, e, i, o, u} y se busca la cantidad de subconjuntos de A
de 3 elementos:

{a, e, i}, {a, e, o}, {a, e, u}, {a, i, o}, {a, i, u}, {a, o, u}, {e, i, o}, {e, i, u}, {e, o, u}, {i, o, u}

En total resultaron 10 por lo que C53 = 10. Observe que para formar un subconjunto de 3 elementos
de un conjunto de 5 elementos, pues lo que se hace es escoger cuáles son los tres que conformarán
el subconjunto, y esto significa que al mismo tiempo se está no escogiendo a los restantes 2, por
lo tanto, por cada subconjunto de 3 elementos sı́ escogidos existe uno y sólo un subconjunto de no
escogidos, y viceversa; es decir, por el principio de biyección C53 = C52 . A continuación se muestran
los subconjuntos de elementos no escogidos de la sucesión de subconjuntos anteriores:

{o, u}, {i, u}, {i, o}, {e, u}, {e, o}, {e, i}, {a, u}, {a, o}, {a, i}, {a, e}

Con este razonamiento y sin necesidad de construir todos los subconjuntos, si tenemos un conjunto
de cardinalidad 9 se tendrı́a que: C90 = C99 , C91 = C98 , C92 = C97 , C93 = C96 , C94 = C95 ¿Observa cuál es
el patrón?

Teorema 2.1. Para todo n ≥ k se cumple la siguiente identidad combinatoria

Cnk = Cnn−k
7
La primera notación utilizada puede tener invertido el orden de los parámetros dependiendo el autor que se
consulte.

18
2.1 MODELO DE CONJUNTOS 2 Combinaciones y el Número Combinatorio

Demostración. Dado que en un conjunto de n elementos, por cada subconjunto de k elementos


existe uno y sólo un subconjunto de n − k elementos (una vez escogidos k, los n − k son los no
escogidos), y viceversa, entonces contar los subconjuntos de k elementos da el mismo resultado que
contar los subconjuntos de n − k elementos, y de allı́ se sigue el resultado.

EJEMPLO 2.2
Un combinatorio fácil de calcular es Cn1 , es decir, la cantidad de subonjuntos unitarios de un
conjunto de n elementos, claramente Cn1 = n, y por la identidad anterior también se cum-
plirá Cnn−1 = n.

EJEMPLO 2.3
Calculemos Cn2 .

Se trata por supuesto de elegir una pareja de elementos del conjunto que tiene n elementos. Para
calcularlo podemos imaginar ordenados los elementos del conjunto dado. Consideremos primera-
mente las parejas que incluyen el primer elemento; éste puede combinarse para formar la pareja con
cualesquiera de los (n − 1) elementos restantes, en consecuencia hay (n − 1) parejas que incluyen
el primer elemento. En seguida consideremos las parejas que no incluyen el primer elemento pero
que incluyen el segundo elemento que son en total (n − 2), ya que estas son las posibilidades de
combinar el segundo elemento con cada uno de los elementos restantes. Las parejas que no incluyen
ni el primero, ni el segundo elemento, pero sı́ el tercer elemento, son en número igual (n − 3), y
ası́ sucesivamente hasta llegar a considerar las parejas que no incluyen a ninguno de los (n − 2)
primeros elementos, pero sı́ el penúltimo, que únicamente puede ser combinado con el último,
dando ası́ sólo 1 posibilidad. El análisis anterior nos permite afirmar que el número Cn2 es igual
a la suma de los números naturales desde el 1 hasta el (n − 1), cuyo resultado en representación
compacta de sumatoria es:
n
X n(n − 1)
Cn2 = i=
i=1
2

El método de análisis utilizado para calcular Cn2 , lleva implı́cito un método para generar las dife-
rentes alternativas de formar los subconjuntos deseados, problema que con frecuencia es tan o más
importante que saber determinar el número de posibilidades. Sin embargo, cuando de determinar
sólo el número de alternativas se trata, el método de conteo anterior no resulta tan cómodo y es
preciso obtener otras propiedades que muestran un poco más su lógica interna y que eventualmente
simplifican los cálculos.

Por ejemplo, ya se demostró que la cantidad de subconjuntos de un conjunto de n elementos es 2n ;


podemos recalcular este número dividiendo los subconjuntos en clases disjuntas: los subconjuntos
de cardinalidad 0, los subconjuntos de cardinalidad 1, los subconjuntos de cardinalidad 2, ..., los
subconjuntos de cardinalidad n − 1, los subconjuntos de cardinalidad n. Dado que la cantidad
de subconjuntos de cada una de estas clases me la da un número combinatorio, respectivamente
Cn0 , Cn1 , Cn2 , . . . , Cnn−1 , Cnn , y por el principio de la suma, se tiene demostrado el siguiente teorema:

19
2.2 MODELO DE CAMINOS 2 Combinaciones y el Número Combinatorio

Teorema 2.2. Para todo n se cumple la siguiente identidad combinatoria


n
X
2n = Cn0 + Cn1 + Cn2 + · · · + Cnn−1 + Cnn = Cnk
k=0

Este analisis realmente es fructı́fero, podemos demostrar también este otro resultado

Teorema 2.3. Identidad de Pascal: Para todo n y para todo k ≤ n se cumple la siguiente identidad
combinatoria
Cnk = Cn−1
k−1 k
+ Cn−1

Demostración. Si deseamos calcular Cnk y consideramos un elemento en particular, los dos posibles
casos (disjuntos) son: que se incluya o no éste elemento en los subconjuntos por formar. Si el
elemento se incluye, únicamente hace falta seleccionar (k − 1) elementos de los (n − 1) restantes
k−1
para completar los k, y el total de formas de hacer esto es Cn−1 ; si no se incluye hará falta
k
seleccionar los k elementos de los n − 1 elementos restantes, es decir, hay Cn−1 posibilidades en
este caso. Luego, el teorema queda demostrado por el principio de la suma.
Una de las técnicas básicas de conteo consiste en contar de dos maneras distintas un conjunto de
objetos, lo que permite establecer algunas identidades. He aquı́ un ejemplo tı́pico:

EJEMPLO 2.4
Supongamos que deseamos seleccionar una directiva de k personas en una asamblea de n personas
y en la directiva debe tenerse 1 presidente.

Si consideramos el problema de determinar el número de alternativas que se tienen para realizarlo,


éste podemos resolverlo en las dos formas siguientes: primero seleccionamos los k directivos y
en seguida seleccionamos el presidente entre los k directivos; o bien, primero seleccionamos el
presidente de entre los n asambleistas y en seguida seleccionamos los k − 1 directivos restantes
de entre los n − 1 asambleistas restantes. Al realizarlo de la primera de las formas el número de
k−1
alternativas es Cnk Ck1 ; en la otra forma indicada, se obtiene Cn1 Cn−1 . Ası́, hemos logrado establecer
las relaciones:

Cnk Ck1 = Cn1 Cn−1


k−1

kCnk = nCn−1
k−1

2.2. MODELO DE CAMINOS


Definición 2.2. Puntos Reticulares: Llamaremos puntos reticulares o puntos látices a puntos (a, b)
en el plano cartesiano tales que a, b ∈ Z.

Considere ahora los puntos reticulares A(0, 0) y B(k, n − k) ubicados en el primer cuadrante del
plano, estos definen una retı́cula (o malla) tomando estos puntos como esquinas opuestas, y dicha
retı́cula formada por cuadraditos de lado 1. ¿Cuántos caminos sobre la retı́cula de longitud mı́nima

20
2.2 MODELO DE CAMINOS 2 Combinaciones y el Número Combinatorio

hay de A a B?

Observe que para que los caminos sean de longitud mı́nima, los movimientos permitidos sólo son
hacia la derecha → y hacia arriba ↑; ası́, en total, hay que avanzar k unidades hacia la derecha y
n − k hacia arriba. Entonces, el problema planteado lo puedo convertir en un problema de decisio-
nes: me muevo de punto reticular a punto reticular, hay que avanzar n unidades, avanzando unidad
por unidad y a cada paso decidiendo si es hacia la derecha o hacia arriba; pero k decisiones son
obligatoriamente hacia la derecha, y dependiendo cuáles de estas decisiones sean hacia la derecha
se me definirá un camino (el resto n − k decisiones forzosamente tendrán que ser hacia arriba),
y viceversa. Ahora bien, del conjunto de las n decisiones, la cantidad de formas de escoger k pa-
ra que sean movimientos hacia la derecha es Cnk , por lo tanto, el número de caminos buscado es Cnk .

Teorema 2.4. El total de caminos sobre la retı́cula definida por A(0, 0) y B(k, n−k), y de longitud
mı́nima, es igual a Cnk

B(k, n − k)
...

.. . ..
. .. .

...
A(0, 0)

Un detalle interesante es que para llegar al punto B(k, n − k) existe únicamente dos formas: o se
llega por el punto reticular B1 (k − 1, n − k), o bien por B2 (k, n − k − 1); además, observe que
estos casos son excluyentes, porque un camino de A a B que pase por B1 y B2 tendrá que tener al
menos un movimiento hacia la izquierda o al menos un movimiento hacia abajo, por lo que ya no
serı́a un camino de longitud mı́nima.

Caso 1: El total de formas de llegar de A a B pasando por B1 (cumpliendo las exigencias, que
sean caminos de longitud mı́nima utilizando únicamente la retı́cula definida por A y B) es, por
el principio de la multiplicación, el total de formas de llegar de A a B1 por el total de formas de
llegar de B1 a B. Dado que para llegar de A a B1 se toman en total n − 1 decisiones, de las cuales
k−1
k − 1 son hacia la derecha, el total de caminos de longitud mı́nima de A a B1 es Cn−1 ; además,
como el total de formas de llegar de B1 a B utilizando caminos de longitud mı́nima es 1, se tiene
k−1 k−1
que el total de formas de llegar de A a B pasando por B1 es Cn−1 · 1 = Cn−1 .

Caso 2: Este se resuelve de forma muy similar al caso 1, el total de formas de llegar a A a B
pasando por B2 es igual al producto de la cantidad de formas de llegar de A a B2 por la cantidad
de formas de llegar de B2 a B. Para llegar de A a B2 , en total se toman n − 1 decisiones, de las

21
2.2 MODELO DE CAMINOS 2 Combinaciones y el Número Combinatorio

k
cuales k son hacia la derecha, por lo que el total de caminos de A a B2 es Cn−1 . De aquı́ se concluye
k
fácilmente que el total de caminos del caso 2 es Cn−1 .

Finalmente, por el principio de la suma, dado que estos casos son excluyentes, el total de caminos
de A a B es igual al total de caminos de A a B pasando por B1 más el total de caminos de A a B
pasando por B2 , es decir

Cnk = Cn−1
k−1 k
+ Cn−1
Se obtuvo de nuevo la Identidad de Pascal (2.3). Esta identidad proporciona un algoritmo (no
muy eficiente) para calcular el número combinatorio de forma recursiva, ya que expresa el número
combinatorio de subı́ndice n en términos de combinatorios de subı́ndices n − 1, que a su vez
podrı́an escribirse en términos de combinatorios de subı́ndies n − 2, y ası́ sucesivamente hasta
llegar a combinatorios de fácil cálculo. A continuación mostramos una tabla

k 0 1 2 3 4 5 6 7 8 9
n
0 1
1 1 1
2 1 2 1
3 1 3 3 1
4 1 4 6 4 1
5 1 5 10 10 5 1
6 1 6 15 20 15 6 1
7 1 7 21 35 35 21 7 1
8 1 8 28 56 70 56 28 8 1
9 1 9 36 84 126 126 84 36 9 1

Este modelo del número combinatorio es independiente del modelo de conjuntos, observe que sin
mucho esfuerzo pueden hacerse las deducciones Cn0 = 1 = Cnn , Cn1 = n y Cnk = Cnn−k . ¡Inténtelo!
De mayor reto es por ejemplo demostrar por este método el teorema (2.2); este lo podemos analizar
de la siguiente forma: considere la siguiente cuadrı́cula, el total de caminos de longitud n que
parten de A(0, 0) formados sólo con movimientos → y ↑ puede contarse de dos formas distintas, la
primera por el principio de la multiplicación, como en cada uno de los n movimientos se tienen las
dos opciones, en total son 2n caminos; la segunda es dividir estos caminos en casos excluyentes,
los caminos que terminan en B0 (0, n), los que terminan en B1 (1, n − 1), y ası́ sucesivamente, hasta
contar los que terminan en Bn (n, 0)

22
2.3 CADENAS DE CEROS Y UNOS 2 Combinaciones y el Número Combinatorio

B0 (0, n)

B1 (1, n − 1)

B2 (2, n − 2)

A(0, 0) Bn (n, 0)

Estos conteos devuelven Cn0 , Cn1 , . . . , Cnn , respectivamente, y de allı́ que

2n = Cn0 + Cn1 · · · + Cnn

2.3. CADENAS DE CEROS Y UNOS


Muchos problemas combinatorios, mediante la biyección adecuada, pueden transformarse a pro-
blemas de cadenas de ceros y unos, el número combinatario por sı́ mismo es un ejemplo de ello.
Teorema 2.5. La cantidad de cadenas de k ceros y n − k unos es Cnk .
Demostración. Hasta el momento se ha dicho que Cnk pude interpretarse 1) Como la cantidad
de subconjuntos de k elementos de un conjunto de cardinalidad n, o lo que es lo mismo, es la
cantidad de formas de escoger k objetos de entre n objetos diferentes; y 2) representa la canti-
dad de caminos de longitud mı́nima (sobre la malla) que van del punto A(0, 0) al punto B(k, n−k).

Ahora bien, ambas interpretaciones del número combinatorio pueden asociarse a un tercer proble-
ma: ¿Cuántas cadenas de ceros y unos se pueden formar con k ceros y n − k unos?

Una cadena de k ceros y n − k unos puede interpretarse de la siguiente forma: Dado el conjunto
A = {x1 , x2 , . . . , xn }, cada elemento puede o no ser escogido para conformar a un subconjunto; si
denotamos por 0 a “sı́ se escoge” y por 1 a “no se escoge”, una cadena de k ceros y n − k unos
representa la sı́ escogitación de k elementos de A y la no escogitación de los restantes n − k, es
decir, hace referencia la conformación de un subconjunto de k elementos de A; además, la posición
de los ceros hace referencia a qué elemento de A se escogió; por ejemplo

011
| {z· · · 0}
n

representa que sı́ se escoge a x1 , que no se escogen x2 y x3 ,..., que sı́ escoge xn . El orden entre los
ceros y unos puede cambiar, pero la cantidad de ceros debe ser k.

Con los caminos, la biyección es aún más evidente: a un movimiento a la derecha (avanza una
unidad horizontalmente) se le asocia un 0, en total son k de estos movimientos, y a un movimiento
hacia arriba se le asocia un 1, que son en total n − k; por tanto, la cantidad de cadenas con k ceros

23
2.4 TRIÁNGULO DE PASCAL 2 Combinaciones y el Número Combinatorio

y n − k unos es igual a la de caminos de longitud mı́nima desde A(0, 0) a B(k, n − k), que son en
total Cnk .

Finalmente, se puede dar una aproximación al problema de las cadenas de k ceros y n − k unos de
forma directa: puedo interpretar esto como que si se tienen a disposición n espacios vacı́os en los
que se colocará o bien un 0 o bien un 1; además, se tiene prefijada la cantidad de ceros y unos que
deben colocarse en esos n espacios, en total son exactamente k ceros y obviamente los restantes
n − k deben ser unos. Está claro que si se colocan de alguna manera los k ceros, las posiciones
que deben ocupar entonces los unos están ya fijadas y no hay nada más que hacer, entonces, me
basta conocer de cuántas formas puedo colocar los ceros y luego, las posiciones de los unos quedan
determinadas; pero este conteo es familiar: ¿De cuántas formas puede escoger k espacios vacı́os
(para colocar los ceros) de los n espacios vacı́os disponibles?... pues de Cnk formas.

2.4. TRIÁNGULO DE PASCAL


La siguiente pirámide de números es conocida como Triángulo de Pascal

1 1

1 2 1

1 3 3 1

1 4 6 4 1

1 5 10 10 5 1

1 6 15 20 15 6 1

1 7 21 35 35 21 7 1

1 8 28 56 70 56 28 8 1

1 9 36 84 126 126 84 36 9 1
..
.
Cada fila está formada por la anterior, cada número es igual a la suma de los dos números que se
ubican ligeramente a la izquierda y a la derecha de la fila anterior, y el número de la primera fila
es 1; en caso de que sólo tenga uno de esos números, se considera igual a cero al que no aparece
en la pirámide.

Teorema 2.6. Los números del triángulo de Pascal son números combinatorios.

Demostración. Dado que el primer número del triángulo es 1, la forma de construcción y la iden-
tidad de Pascal (2.3), el teorema queda demostrado.

24
2.5 BINOMIO DE NEWTON 2 Combinaciones y el Número Combinatorio

2.5. BINOMIO DE NEWTON


Considere la expresión
n veces
z }| {
(x + y)n = (x + y)(x + y) · · · (x + y)
Si se efectúan todos los productos aparecerán los términos xn , xn−1 y, xn−2 y 2 , . . . , xy n−1 , y n con sus
respectivos coeficientes; entonces la pregunta es ¿cuál es la relación que determina el coeficiente de
cada término xk y n−k ? ¿Esta relación depende de n, de k o de ambos? Analicemos algunos casos
particulares:

(x + y)0 = 1
(x + y)1 = x+y
(x + y)2 = x2 + 2xy + y 2
(x + y)3 = x3 + 3x2 y + 3xy 3 + y 3
(x + y)4 = x4 + 4x3 y + 6x2 y 2 + 4xy 3 + y 4
(x + y)5 = x5 + 5x4 y + 10x3 y 2 + 10x2 y 3 + 5xy 4 + y 5
..
.

Si observamos los coeficientes, aparece la siguiente secuencia, ¿la reconoce?

1
1 1
1 2 1
1 3 3 1
1 4 6 4 1
1 5 10 10 5 1
..
.
¡Claro!, vuelven a aparecer los combinatorios, se obtiene el Triángulo de Pascal. Ahora la cuestión
es justificar por qué los coeficientes del desarrollo (x + y)n son números combinatorios. Lo primero
que hay que hacer es entender cómo se obtiene un coeficiente en particuar al desarrollar (x + y)n .
Veamos un ejemplo particular:

EJEMPLO 2.5
Considere (x + y)4 = (x + y)(x + y)(x + y)(x + y), para desarrollarlo, de cada paréntesis debemos
escoger la x o la y, y luego se multiplican esos cuatro números, al recorrer todas las posibles esco-
gitaciones se obtienen todos los términos, que luego, se suman términos semejantes. Veamos esto
en acción:

Se escribirán las escogitaciones ası́: xxyy representa escoger del primer paréntesis la x, del segundo
paréntesis la x, del tercer paréntesis la y, del cuarto paréntesis la y. Entonces, las posibles esco-
gitaciones son, por el principio de la multiplicación, 25 = 32 (dos opciones por cada paréntesis);
listadas una a una se obtiene

25
2.5 BINOMIO DE NEWTON 2 Combinaciones y el Número Combinatorio

xxxx = x4 xyxx = x3 y yxxx = x3 y yyxx = x2 y 2


xxxy = x3 y xyxy = x2 y 2 yxxy = x2 y 2 yyxy = xy 3
xxyx = x3 y xyyx = x2 y 2 yxyx = x2 y 2 yyyx = xy 3
xxyy = x2 y 2 xyyy = xy 3 yxyy = xy 3 yyyy = y4
Luego, el coeficiente representa la cantidad de veces que aparece el mismo término, por ejemplo,
observe que el término x2 y 2 aparece 6 veces, entonces en el desarrollo de (x + y)4 uno de los suman-
dos es 6x2 y 2 . Centremos nuestra atención en este término, hay que buscar un método que permita
calcular ese 6 de forma sistemática, quizás esto dé una pista para abordar el caso general:

Para que aparezca el término x2 y 2 , se necesita claramente un par de x y un par de y, pero una
vez que se escoge el par de paréntesis de los cuáles se selecciona la x, está claro que de los otros
dos paréntesis deberá escogerse la y; ası́, el total de forma de escogerse 2 paréntesis de los 4 es
C42 = 6.

Exactamente los mismo sucede en el caso general, dado (x + y)n , las veces que aparecerá el término
xk y n−k es igual a la cantidad de formas de escoger k paréntesis de los n (en cada uno de estos
paréntesis se seleccionará la x, en los n − k se seleccionará la y), y esto es Cnk . Ası́, se obtiene la
siguiente identidad algebraica llamada el Binomio de Newton:
Teorema 2.7. Binomio de Newton: El desarrollo del binomio (x + y)n es
n
X
n
(x + y) = Cnn xn + Cnn−1 xn−1 y + Cnn−2 xn−2 y 2 + ··· + Cn1 xy n−1 + Cn0 y n = Cnk xk y n−k
k=0

Esta identidad es muy útil para encontrar y demostrar identidades combinatorias, observe por
ejemplo

EJEMPLO 2.6
Si x = y = 1:

2n = (1 + 1)n = Cnn + Cnn−1 + Cnn−2 + · · · + Cn1 + Cn0

Nos encontramos de nuevo con (2.2 ), que ya se derivó a partir de los subconjuntos de un conjunto
de cardinalidad n, utilizando caminos y ahora con el binomio de Newton. Otro ejemplo

EJEMPLO 2.7
Suponga que x = 1, y = −1, se tiene entonces

(1 − 1)n = Cnn − Cnn−1 + Cnn−2 − Cnn−3 + · · · + (−1)n−k Cnk + · · · + (−1)n Cn0


X X
Cnk = Cnk
k par k impar

Esta es una relación cuya deducción por los métodos anteriores es tremandamente difı́cil; con la
identidad de Newton se puede ir muy lejos, incluso, es posible evaluar en x o en y valores complejos.

26
2.6 PROBLEMAS 2 Combinaciones y el Número Combinatorio

2.6. PROBLEMAS
1. ¿De cuántas formas pueden 4 niñas y 4 niños ser dividos en dos equipos de 4 si cada equipo
debe tener al menos a una niña?

2. ¿De cuántas formas pueden 5 niñas y 3 niños ser dividos en dos equipos de 4 si cada equipo
debe tener al menos a un niño?

3. En una rifa participan 100 personas, regalándose 3 televisores idénticos. Quién diseñó la rifo
pensó que sacar un ticket, luego el segundo y finalmente el tercero era injusto, por lo que se
sacan al mismo tiempo los 3 tickets. ¿Cuál es la probabilidad que una persona que compró un
solo ticket gane?

4. Una asamblea de 14 personas desea elegir entre sus miembros un presidente, un vicepresidente
y un secretario de actas. ¿De cuántas formas puede realizarse la elección?

5. Ocho promotores deben visitar 4 comercios. Para ello forman 4 parejas, debiendo cada una
de ellas visitar un establecimiento. ¿De cuántas formas pueden distribuirse el trabajo?

6. Adrián tiene nueve pares distintos de calcetines, un dı́a se levantó tarde para su clase de
las 9:00AM y en la prisa tomó aleatoreamente ocho calcetines sin mirarlos. ¿Cuál es la
probabilidad de que entre los calcetines tomados hayan exactamente dos pares correctos?

7. Se dispone de una colección de 30 pelotas divididas en 5 tamaños distintos y 6 colores


diferentes, de tal manera que en cada tamaño hay pelotas de todos los colores. ¿Cuántas
colecciones de 4 pelotas tienen exactamente 2 pares de pelotas del mismo tamaño (que no
sean las 4 del mismo tamaño)?

8. En un curso de Combinatoria hay 30 estudiantes, de los cuales 19 son mujeres. Para organizar
una feria de ciencias, se pide que de este curso se presenten tres exposiciones, realizadas por
parejas de estudiantes, tal que cada pareja está intengrada por estudiantes de distinto género,
y un estudiante no puede estar en dos o más parejas. ¿De cuántas formas puede hacerse esto?

9. Se desea elegir una directiva, hay diez candidatos (cinco mujeres y cinco hombres) para los
cargos de presidente, vicepresidente, secretario, tesorero y vocal. ¿De cuántas formas pueden
elegirse los cargos? ¿De cuántas formas es posible hacerlo si una mujer será la presidenta?
¿De cuántas maneras es posible si el tesorero ya está definido que será Juan?

10. Una persona tiene seis amigos. Cada noche, durante cinco dı́as, invita a cenar a un grupo
de tres de ellos, de modo que el mismo grupo no sea invitado dos veces. ¿De cuántas formas
puede hacerlo?

11. Dados los conjuntos A = {1, 2, . . . , 6} y B = {1, 2, . . . , 10}, ¿cuántas funciones estrictamente
crecientes8 f : A → B pueden definirse? ¿cuántas además cumplen que f (4) = 7?

12. Se dice que una mano de dominó (compuesta por 7 fichas de dominó) tiene “falla” si alguno
de los números entre el 0 y el 6 no aparece en la mano. Determine el número de manos de
dominó que no tienen falla.
8
Se dice que f es estrictamente creciente si a < b ⇒ f (a) < f (b).

27
2.6 PROBLEMAS 2 Combinaciones y el Número Combinatorio

13. En cada subconjunto de 7 elementos del conjunto {1, 2, . . . , 10} se escoje el elemento mayor.
¿Cuál es la suma de todos los elementos mayores?

14. Diez puntos están marcados en el plano, no habiendo tres colineales. ¿Cuántos triángulos
pueden formarse?

15. ¿Cuántos cuadriláteros (convexos o no convexos) pueden formarse con los vértices de un
n−ágono regular?

16. Dados seis puntos sobre una circunferencia, se trazan todas las cuerdas que estos puntos
definen. ¿Cuál es la probabilidad de que al escoger aleatoreamente cuatro de estas cuerdas
se forme un cuadrilátero convexo?

17. De cierto número de rectas coplanares se sabe que no hay tres de ellas que concurran en el
mismo punto y no hay ninguna pareja de rectas paralelas. Esas rectas producen 45 puntos
al cortarse. ¿De cuántas rectas estamos hablando?

18. ¿Cuántos puntos de intersección producen 8 rectas coplanares, sabiendo que dos de ellas son
paralelas y no hay tres concurrentes?

19. Se tienen nueve puntos en un plano. Cuatro de ellos están alineados y los restantes están
dispuestos de forma que no hay nunca 3 alineados. ¿Cuántos triángulos pueden formarse que
tengan sus vértices sobre esos 9 puntos? ¿Cuántas rectas distintas determinan esos puntos?

20. En una fábrica hay varios centros de almacenamiento, cada uno de los cuales está unido a
los demás por una cinta transportadora. Calcula el número de centros de la fábrica si se sabe
que el número de cintas transportadoras es 66.

21. Dibuja una circunferencia y marca sobre la misma diez puntos. Uniendo parejas de esos
puntos ¿Cuántos pentágonos convexos distintos se podrı́an formar?

22. Dado el conjunto de dı́gitos {1, 3, 6, 7, 9}, determine el número de maneras de formar números
de 4 cifras tales que sean múltiplos de 3.

23. ¿Cuántos números de cuatro cifras cumplen la propiedad de que el producto de dichas cifras
es un cuadrado perfecto?

24. Un cuadrado de lado 6 es dividido en 36 cuadrados de lado 1. Los puntos A y B son puntos
medios de un par de lados opuestos del cuadrado. A cada lado de la lı́nea AB seis cuadrados
unitarios son tomados aleatoriamente y coloreados de azul. Si el cuadrado grande es doblado
sobre recta AB, calcule la probabilidad de qué exactamente un par de cuadrados azules
coincidan.

25. ¿De cuántas maneras puede distribuirse 3n objetos distintos en tres cajas distintas de modo
que cada caja tenga el mismo número de objetos?

26. Calcule cuántas cadenas de ceros y unos de longitud 7:

a) contienen exactamente 3 unos y 4 ceros.


b) contienen 4 unos consecutivos.

28
2.6 PROBLEMAS 2 Combinaciones y el Número Combinatorio

c) contienen como máximo 3 unos.


d) no contienen 4 o más unos consecutivos.
k−1
27. Demuestre la identidad de Pascal Cnk = Cn−1
k
+ Cn−1 utilizando distintos modelos combina-
torios.
k−1 k−2
k
28. Demuestre que Cnk = C20 Cn−2 + C21 Cn−2 + C22 Cn−2 .
k−1 k−1 k−1
29. Demuestre Cnk = Cn−1 + Cn−2 + · · · + Ck−1 es válido para todo k ≤ n.

30. Demuestre que


2n−1 k
X C2n
=0
k=0
(−2)k

31. Demuestre n
X
3r Cnr = 4n
r=0

32. Hallar el número de caminos crecientes que empiezan en (0, 0) y terminan en alguno de los
puntos indicados.

33. Demuestre por diversos métodos la identidad de Vandermonde


0
Cm Cnr + Cm
1
Cnr−1 + · · · + Cm
r
Cn0 = Cm+n
r

34. En la cuadrı́cula de abajo

B(10, 10)

X(6, 4)

A(0, 0)

a) ¿De cuántas maneras podemos ir de la casilla A a la casilla B con movimientos siempre


hacia la derecha o bien una casilla hacia arriba?
b) ¿De cuántas maneras se puede hacer esto si debemos pasar además por la casilla X?
c) ¿Cuántos caminos llevan de A a B sin pasar por ningún punto de última lı́nea vertical
(salvo B naturalmente)

29
2.6 PROBLEMAS 2 Combinaciones y el Número Combinatorio

d) ¿De cuántas maneras se llega de A a B pasando por un solo punto de la segunda lı́nea
horizontal?

35. Demuestre por diversos métodos la siguiente identidad


n
n
X 2
C2n = Cnj
j=0

36. Dentro de un paralelepı́pedo rectangular de alambre A de dimensiones 5 × 4 × 6 de colocan


alambre dividiendo a A en cubos de lado 1. ¿Cuántos caminos diferentes de longitud mı́nima
hay desde el vértice inferior izquierdo de la cara anterior de A hasta el vértice superior
derecho de la cara posterior de A?

37. Suponga que tiene una armazón cúbica de alambre de l × m × n (la armazón está formada
por cubitos de alambre lado 1). Si una hormiga está ubicada en una esquina y quiere llegar
caminando por los alambres a la esquina más alejada con el mı́nimo recorrido ¿de cuántas
maneras puede hacerlo? ¿y de cuántas formas podrı́a hacerlo si la hormiga camina sobre una
armazón 4−dimensional de l × m × n × k? ¿puede generalizar para más dimensiones?

38. Dado un polı́gono conexo tal que no tiene tres diagonales concurrentes, ¿cuántos puntos de
intersección al interior del polı́gono forman las diagonales?

39. Demuestre la identidad


q−1 1 q q
Cpq Cr0 + Cp−1 0
Cr+1 + · · · + Cp−q Cr+q = Cp+r+1

p−r
40. Demuestre que Cnp Cpr = Cnr Cn−r siempre que n ≥ p ≥ r.

41. Calcule n  
X 1
k+ Cnk
k=1
k

42. Demuestre que


n
X j 1 2n n 2

C2n = C4n + (C2n )
j=0
2

43. Demuestre las identidades combinatorias de Chu Shih Chieh

Crr + Cr+1
r
+ · · · + Cnr = Cn+1
r+1

Cr0 + Cr+1
1 k
+ · · · + Cr+k k
= Cr+k+1

44. Demuestre la identidad


k 2 = 2Ck2 + Ck1
y a partir de ella demuestre que
 
2 2 1 12
1 + 2 + ··· + n = n n + (n + 1)
3 2

30
2.6 PROBLEMAS 2 Combinaciones y el Número Combinatorio

45. Demostrar que para todo natural n se cumple


 
n+1 2n + 1 n
C2(n+1) = 2 C2n
n+1

46. Resuelva las siguiente ecuaciones combinatorias:

a)
2 2 2
Cm−1 + Cm + Cm+1 = 19
b)
x−1 x
7C2x−2 = 2C2x
c)
x2
Cx0 + Cx1 · Cx2 = +2
2
d)
n3 − 6n2 + 20
Cn3 − Cn2 =
6
e)
Cn4 = 13Cn2
f)
1 2 3
C2n + C2n + C2n = 387n
g)
n−5 n−7
Cn−1 = Cn−3
523 524
47. Calcule el valor de C525 + C525
67
20x3 y 2

48. En la expansión de + , determinar:
y5 x
a) El vigésimo término
b) El coeficiente del término cuarenta y cinco
c) El coeficiente de x23 , si es posible. Si no, argumente el por qué no existe.
d) ¿Cuántos términos tiene la expansión?

49. Encuentre el coeficiente de x5 en la expansión de (1 + x + x2 )8 + (1 + x + x2 )9 .


 5
1 6 7
50. En la expansión de x + 3 + x + x encontrar el coeficiente de x4 .
x
51. Encuentre el coeficiente del término a7 b4 ce2 en el desarrollo de (a + b + c + d + e)14 .
12!
52. ¿Cuál es la suma de todos los números de la forma si a, b, c varı́an sobre todos los
a!b!c!
enteros no negativos que cumplen que a + b + c = 12?

31
2.6 PROBLEMAS 2 Combinaciones y el Número Combinatorio

1 25
53. Dada la expresión (7x2 + 2x ) , encuentre la cantidad de términos de su expansión, la suma
de los coeficientes de su expansión, el coeficinte de x1 3 si es posible, y encuentre el i-ésimo
término.
k−1 k+1 k k k+1
54. Demuestre la identidad del hexágono: Cn−1 Cn Cn+1 = Cn−1 Cnk−1 Cn+1 . El nombre viene del
hecho que los coeficientes binomiales implicados forman una figura hexagonal alrededor de
Cnk en el triángulo de Pascal.

55. Encuentre el coeficiente de xn y de xn+r (con 1 ≤ r ≤ n) en la expansión de

(1 + x)2n + x(1 + x)2n−1 + x2 (1 + x)2n−2 + · · · + xn (1 + x)n

56. ¿Cuántos paralelogramos quedan determinados cuando un grupo de 6 rectas paralelas es


intersecado por otro grupo de 6 rectas paralelas?

57. Un tablero de 5 × 5 es dividido en 25 cuadrados unitarios, de estos dos son pintados de azul y
el resto pintado de blanco, diremos que dos coloraciones son iguales si una puede ser obtenida
de la otra al rotar tablero. ¿Cuántas coloraciones distintas existen?

58. Un polı́gono convexo de n lados es tal que no hay punto común alguno para cualesquiera tres
de sus diagonales. Determine el número de triángulos que se forman de manera tal que dos
de sus vértices sean vértices del polı́gono y el tercero sea una interseccción de dos diagonales.

59. Una rana se ubica en el tercer escalón de unas gradas, la rana se mueve un escalón por salto.
¿Cuántas formas existen para que la rana llegue por primera vez al octavo escalón en su
noveno salto?

60. ¿De cuántas formas pueden ordenarse los enteros del 1 al n bajo la siguiente condición:
excepto por el primer entero de la izquierda, todo entero debe diferir por 1 de algún entero
que esté más la izquierda que este?

32
3 Permutaciones y Arreglos

3. Permutaciones y Arreglos
3.1. PERMUTACIONES
Sea A un conjunto finito. Ordenar elementos de A es darle a cada elemento del conjunto una posi-
ción determinada, es decir definir qué elemento ocupa la primera posición, el que ocupa la segunda
posición y ası́ sucesivamente. Dos ordenamientos de elementos de A se dirá que son idénticos si
todos los elementos en ambos ordenamientos se encuentran en la misma posición; en consecuencia
dos ordenamientos serán diferentes si difieren en la posición en la que se encuentra alguno de los
elementos. Hay diversas formas de ordenar los elementos de un conjunto y en este apartado nos
ocuparemos de contar el total de alternativas de ordenamiento de los elementos de un conjunto de
cardinalidad n.

EJEMPLO 3.1
¿De cuántas formas pueden ordenarse los elementos del conjunto A = {a, b, c}?

Los elementos del conjunto A = {a, b, c} se pueden ordenar de las siguientes seis formas: abc, acb, bac, bca, cab
y cba. Observe que para contar los posibles ordenamientos nos basta definir el elemento que ocu-
pará la primera posición, el que ocupará la segunda y el de la tercera posición, lo que es equivalente
a contar las ternas (x, y, z) de elementos de A con la condición que los elementos de la terna sean
todos diferentes. En este caso, la primera posición puede ser ocupada por uno cualesquiera de los
elementos de A, es decir contamos en este caso 3 posibilidades; para la segunda posición ya sólo
disponemos de 2 posibilidades y para la posición tercera sólo hay 1 posibilidad.

En general cuando el conjunto A tiene n elementos, el número total de formas de ordenarlos es n!


puesto que la primera posición puede ser ocupada por uno cualesquiera de los n elementos de A;
la segunda posición por cualquier elemento que no sea el colocado en la primera posición, por lo
que las alternativas se reducen a (n − 1); para la tercera posición se tienen (n − 2) alternativas
y ası́ sucesivamente dejando a la última posición con una única alternativa, y por el principio de
la multiplicación las alternativas de ordenar los elementos de A serán n(n − 1)(n − 2) · · · (2)(1) =
n!. A cada ordenamiento de los elementos de A se le denomina una permutación, y al total de
permutacione se denota por Pn . Tenemos entonces el resultado siguiente:

Teorema 3.1. El total de permutaciones de n elementos, denotado por Pn es

Pn = n!

3.2. PERMUTACIONES CIRCULARES


Supongamos que debemos colocar a n personas alrededor de una mesa circular en la que se dispone
de n posiciones numeradas para su ubicación y deseamos determinar el total de alternativas de
ordenamiento. Si dos ordenamientos dados los consideramos diferentes cuando las personas ocupan
posiciones numeradas diferentes, entonces el número de posibles ordenamientos será igual a un
ordenamiento en una fila, es decir, Pn ; a estas les llamaremos permutaciones lineales. Sin embargo
puede ser que sólo nos interese la posición relativa que guardan entre sı́ las personas y no nos
interese el número de la posición que ocupan, a estas permutaciones les llamaremos permutaciones

33
3.3 PERMUTACIONES CON REPETICIÓN 3 Permutaciones y Arreglos

circulares; si este es el caso hay n permutaciones lineales que dejarı́an a las personas con la
misma permutación circular, en efecto, la rotación de las personas pasando por las n posiciones
numeradas dejan a las personas en la misma posición relativa. En consecuencia hay n permutaciones
lineales por cada permutación circular; como el total de permutaciones lineales es Pn , el total de
permutaciones circulares, el cual denotamos por Cn , será
Teorema 3.2. El total de permutaciones circulares de n elementos, denotado por Cn es
Pn
Cn = = (n − 1)!
n
Hay otra manera de abordar esta mismo problema: Supongamos que una persona X se sienta
primero, es indistinto qué asiento escoge dado que lo importante es cómo se ubiquen las restantes
n − 1 personas, numeramos 1 al asiento que escogió X y a partir de este se numeran los restantes
asientos (en orden horario o antihorario, no importa); ahora las n − 1 personas restantes pueden
permutarse linealmente de Pn−1 formas, y una vez que han decidido una permutación lineal en par-
ticular, toman los asientos 2, 3, . . . , n en el orden prefijado, eso da nuevamente Cn = Pn−1 = (n−1)!

Ahora considere un problema parecido pero que entre lı́neas lleva una restricción más, suponga
que se tienen 5 objetos distintos y se quieren elaborar todos los collares posibles utilizando estos
cinco objetos, ¿cuántos hay? ¡Inténtelo!

3.3. PERMUTACIONES CON REPETICIÓN


Suponga ahora un problema distinto: se quieren permutar n objetos, pero no todos son distintos,
hay dos clases de objetos, hay k1 objetos idénticos de la primera clase y k2 objetos idénticos de la
segunda clase (obviamente k1 + k2 = n), ¿cuántas permutaciones hay?

EJEMPLO 3.2
¿Cuántas permutaciones se pueden construir con 2 bolas negras idénticas y 2 bolas blancas también
idénticas?

Si las 4 bolas fuera todas distintas, el total de permutaciones (lineales) serı́a 4!; enlistemos todas
las posibilidades, se denotará por n1 y n2 a las bolas negras, y por b1 y b2 las blancas:

n 1 n 2 b1 b2 n1 b1 n2 b2 n1 b1 b2 n2 b1 b2 n 1 n 2 b1 n 1 b2 n 2 b1 n 1 n 2 b2
n 1 n 2 b2 b1 n1 b2 n2 b1 n1 b2 b1 n2 b2 b1 n 1 n 2 b2 n 1 b1 n 2 b2 n 1 n 2 b1
n 2 n 1 b1 b2 n2 b1 n1 b2 n2 b1 b2 n1 b1 b2 n 2 n 1 b1 n 2 b2 n 1 b1 n 2 n 1 b2
n 2 n 1 b2 b1 n2 b2 n1 b1 n2 b2 b1 n1 b2 b1 n 2 n 1 b2 n 2 b1 n 1 b2 n 2 n 1 b1
Ahora, observe que, dado que las dos bolas negras son iguales entre sı́, y las dos bolas blancas
tambien son iguales entre sı́, cada columna, realmente representa a la misma permutación, si les
quitamos los subı́ndices quedarı́a

nnbb nbnb nbbn bbnn bnbn bnnb


nnbb nbnb nbbn bbnn bnbn bnnb
nnbb nbnb nbbn bbnn bnbn bnnb
nnbb nbnb nbbn bbnn bnbn bnnb

34
3.3 PERMUTACIONES CON REPETICIÓN 3 Permutaciones y Arreglos

Entonces, el total de permutaciones distintas bajo estas condiciones no es 4! sino que sólo 6.

¿Cómo podrı́amos calcular este 6 sin necesidad de hacer una por una todas las permutaciones?,
la idea central está allı́, se trata de, en primer lugar, suponer que todos los objetos son distintos,
entonces, el total de permutaciones ya se sabe como calcularlo, es 4!; pero, dado que no todos los
objetos son distintos, hay que ver cuántas veces aparece una verdadera permutación en el con-
teo anterior; tome por ejemplo bnnb, cuando se suponen todos distintos, las letras b se pueden
permutar de 2! formas mientras que las letras n se pueden permutar de otras 2! formas, por lo
que bnnb aparecerá 2!2! = 4 veces cuando consideremos distintos a los objetos (coincide con la
tabla, ¿cierto?). Entonces 4! es el cuádruple de las permutaciones que buscamos, es decir, que las
4!
permutaciones con objetos repetidos son en total = 6.
2!2!
Resolvamos ahora el caso general: se tienen k1 objetos de una clase y k2 objetos de otra clase
(los objetos de una misma clase son idénticos entre sı́), y denotamos por P (k1 , k2 ) al total de
permutaciones de estos objetos. Tomando n = k1 + k2 , si todos los objetos fueran distintos, el total
de permutaciones serı́a Pn , pero en este conteo, cada permutación de las buscadas aparece repetida
k1 !k2 ! veces, por lo que

Teorema 3.3. El total de permutaciones con repetición de k1 objetos de un tipo y k2 objetos de


otro tipo, denotado por P (k1 , k2 ) y tomando n = k1 + k2 es
n!
P (k1 , k2 ) =
k1 !k2 !
Este argumento se puede generalizar:

Teorema 3.4. Dados k1 objetos idénticos de una clase 1, luego otros k2 objetos idénticos de una
clase 2, ..., kr objetos idénticos de una clase r, y tomando n = k1 + k2 + · · · + kr , el total de
permutaciones es
n!
P (k1 , k2 , . . . , kr ) =
k1 !k2 ! · · · kr !
Por otra parte, ya antes se habı́a trabajado un problema muy similar a este, el de las cadenas
de longitud n de ceros y unos, todos los ceros son iguales entre sı́, todos los unos también, y se
está interesado en la cantidad de cadenas distintas que se pueden formar con k ceros y n − k unos,
es decir, todas las permutaciones con k objetos de idénticos de una clase y n−k de la otra; además,
ya se demostró que el total de estas cadenas es Cnk , entonces

Teorema 3.5. El número combinatorio puede interpretarse como permutaciones con elementos
repetidos, y se relacionan ası́
n!
Cnk = P (k, n − k) =
k!(n − k)!

Nota: En la relación anterior, observe que pueden darse los casos k = 0 (sólo hay unos) y k = n
(sólo hay ceros), y según los modelos de número combinatorio, Cn0 = 1 = Cnn , por lo que para que
la relación anterior siga siendo válida en estos casos, se define 0! = 1.

35
3.4 ARREGLOS 3 Permutaciones y Arreglos

3.4. ARREGLOS
Regresamos a ordenar objetos distintos, pero considerando que la cantidad de espacios es inferior
o igual a la cantidad de objetos distintos que se ordenan; es decir, ¿cuántas configuraciones de
longitud k puede formarse si se tienen n objetos distintos (k ≤ n)? A estas configuraciones les lla-
maremos arreglos, y al total de arreglos de longitud k dados n objetos distintos se le denotará por
Akn .9

Este problema es una aplicación directa del principio de la mutiplicación, y sin duda podemos
argumentar el caso general sin inconveniente alguno: se tienen k espacios, en el primero puede
ubicarse a cualquiera de los n objetos, hay etonces n opciones; en el segundo espacio puede ubicarse
a cualquiera de los n objetos exceptuando el que se colocó en la primera posición, por lo que hay
n − 1 opciones; en el tercer espacio puede colocarse a cualquiera de los n objetos exceptuando el
que se colocó en la primera posición y el que se colocó en la segunda posición, por lo que hay n − 2
opciones; este argumento se sigue ası́ sucesivamente, hasta que en el espacio k puede ubicarse a
cualquiera de los n objetos, exceptuando los que se colocaron en las anteriores k − 1 posiciones,
por lo que hay n − (k − 1) opciones. Ası́

Teorema 3.6. El total de arreglos de longitud k dados n objetos distintos (k ≤ n) es

Akn = n(n − 1)(n − 2) · · · (n − (k − 1))

Pero esta expresión puede escribirse de forma más compacta multiplicanndo y dividiendo por
(n − k)!

n(n − 1)(n − 2) · · · (n − (k − 1))(n − k)! n!


Akn = =
(n − k)! (n − k)!
En particular si n = k se obtiene
Ann = Pn
Observe que Cnk y Akn cuenta estructuras similares, la diferencia radica que cuando se calcula el
combinatorio, únicamente interesa escoger k objetos de los n disponibles (el orden de los objetos no
importa), en cambio, cuando se calcula la cantidad de arreglos, luego de escogerlos se construyen
las k! permutaciones (el orden sı́ importa), entonces

Teorema 3.7. Dados n objetos distintos, la cantidad de arreglos y de combinaciones de longitud


k (k ≤ n) se relacionan por
Akn = k!Cnk

3.5. PROBLEMAS
1. Para confeccionar un examen, se dispone de 3 problemas de Geometrı́a, 4 de Combinatoria
y 2 de Álgebra. ¿De cuántas maneras pueden ordenarse los problemas si los que corresponde
a un mismo tema deben aparecer en forma consecutiva?
9
La notación más usual para este número es Pnk , y a estas se les llama también permutaciones, pero dado que
permutar significa únicamente cambiar de orden, reservaremos esa palabra exclusivamente para el caso en el que
hay igual número de objetos que de espacios.

36
3.5 PROBLEMAS 3 Permutaciones y Arreglos

2. El equipo de Anita y el de Chepito juegan futbol con la regla adicional siguiente: gana quién
primero obtiene cuatro goles y no hay empates. ¿De cuántas formas puede ganar el equipo
de Anita?

3. ¿De cuántas formas es posible escoger 4 cartas de distinto manjar de una baraja de 52 cartas?

4. Un bebé recién nacido puede tener 1, 2 ó 3 nombres. ¿De cuántas formas puede llamarse si
se puede escoger de 300 nombres disponibles?

5. En una mesa redonda de 5 asientos se sientan 7 personas. ¿De cuántas formas pueden hacerlo
si la persona 1 es enemiga de la persona 2 y si se sienta una no se sienta la otra?

6. ¿De cuántas formas pueden entregarse 6 cartas urgentes si se tienen a disposición 3 couriers
distintos?

7. ¿Cuántas permutaciones hay que no tengan I juntas de la palabra PARANGUATIRIMICUA-


RO?

8. A una conferencia han sido invitadas como exponentes 5 personas: A, B, C, D y E. ¿De


cuántas formas se pueden ordenar las exposiciones si B no debe preceder a A? ¿Cuántas
formas distintas serı́an si B debe hablar inmediatamente después que A?

9. ¿De cuántas formas pueden sentarse 5 mujeres y 5 hombres en una mesa redonda de tal
forma que las personas vecinas de cada quien son de género distinto?

10. ¿Cuál es la cantidad de configuraciones distintas que pueden generar los m semáforos ubicados
sobre una calle principal?

11. Una madre tiene 2 manzanas y 3 peras, y le da a su hija una fruta cada dı́a (de lunes
a viernes), ¿de cuántas formas puede hacerlo? ¿De cuántas formas podrı́a hacerlo si tiene
además 4 naranjas?

12. Una profesora distribuye 5 naranjas distintas entre sus 8 estudiantes de tal forma que cada
estudiante recibe a lo sumo una naranja, ¿de cuántas formas puede hacerlo? ¿Cuántas formas
serı́an si ya no se restringe la cantidad que puede recibir cada estudiante?

13. Un club deportivo de 30 miembros quiere hacer 4 equipos de 4 personas cada uno para que
participen en un rally, ¿de cuántas formas puede hacerlo?

14. ¿De cuántas formas pueden distribuirse m + n + p objetos distintos en 3 cajas tal que cada
una tenga m, n y p objetos respectivamente?

15. Determine el número de permutaciones de la palabra TONACATEPEQUE.

16. Dada una baraja de 4 manjares y 52 cartas (diamantes, corazones, espadas y tréboles, 13
cartas de cada manjar), ¿de cuántas maneras es posible escoger 4 cartas de distinto manjar
tal que el valor de la carta de diamantes sea igual al valor de la de corazones, y el valor de
la carta de espadas sea igual al valor de la de tréboles?

37
3.5 PROBLEMAS 3 Permutaciones y Arreglos

17. Con una baraja igual a la del ejercicio anterior, determine la probabilidad de obtener en la
primera mano de un juego de póker: una escalera de color, un póker, una escalera, color, una
casa llena, un trı́o, dos pares.

18. Si se juega con 6 dados, dos negros, dos verdes y dos rojos, ¿cuál es la probabilidad de obtener
una escalera tal que los dados del mismo color tengan números consecutivos? (considere al 6
y al 1 como consecutivos también)

19. En El Salvador, los números telefónicos se forman con 8 cifras, siendo 2 el primero de ellas,
y la segunda 2, 3, 4, 5 ó 6; en Guatemala en cambio son sólo 7 cifras pero con las mismas
restricciones para sus primeras dos cifras. ¿Cuántas comunicaciones pueden establecerse entre
los habitantes de ambos paı́ses?

20. ¿Cuántos bloques coloreados diferentes, de forma cúbica fija, puede hacerse si hay seis colores
disponibles y cada bloque debe tener un color diferente en cada una de sus seis caras?

21. La familia Pérez ha comprado una mesa circular nueva con seis sillas, justa para toda la
familia. Para esta familia, no hay preferencias con respecto a cuál silla ocupar, sólo les
interesa saber quienes tienen a la par. ¿De cuántas formas pueden sentarse si Juanita y
Pedro están peleados y no quieren sentarse juntos? ¿y si la mamá Pérez quiere sentarse a la
par de Anita?

22. Hallar la cantidad de números que se obtengan como permutaciones del número 111122256
que sean divisibles por 12.

23. Seis matrimonios se reúnen a cenar en una mesa circular. ¿De cuántas formas pueden ubicarse,
si cada hombre debe estar flanqueado por dos mujeres y los miembros de cada pareja deben
estar juntos?

24. Cuatro bailarines y cuatro bailarinas interpretan una danza que consiste en formar una ronda
tomados de la mano. ¿De cuántas formas pueden ubicarse si en la figura deben aparecer
alternadamente hombres y mujeres?

25. Sean k, n ∈ N. Demuestre que el número de formas de sentar kn personas alrededor de k


mesas distintas de modo que hay n personas en cada mesa es

(kn)!
nk

38
4 Extensiones del Número Combinatorio

4. Extensiones del Número Combinatorio


4.1. SEPARADORES
Considere el siguiente problema:

EJEMPLO 4.1
Ana quiere comprar 10 dulces para regalárselos a sus primitos; en la tienda hay dulces de tres
sabores, menta, fresa y limón, ¿de cuántas formas puede escogerlos?

Si llamamos m a la cantidad de dulces de menta, f la cantidad de dulces de fresa y l la cantidad


de dulces de limón, debe cumplirse que m + f + l = 10, y obviamente cada uno de estos números es
mayor o igual a cero. Como siempre, lo mejor es analizar algunos casos particulares, por ejemplo
m = 0, entonces f + l = 10, y las parejas solución son

(f, l) = (0, 10), (1, 9), (2, 8), (3, 7), (4, 6), (5, 5), (6, 4), (7, 3), (8, 2), (9, 1), (10, 0)

en total hay 11 posibilidades. Si analizamos el resto de casos (todos disjuntos) m = 1, 2, . . . , 10,


se obtendrán respectivamente 10, 9, . . . , 1 posibilidades, por lo que la cantidad de formas que Ana
puede hacer la compra es 11 + 10 + · · · + 1 = 66.

Este método resolvió el problema y las cuentas no fueron largas ni difı́ciles, sin embargo no nos da
idea de como abordar un problema con más variables o con números más grandes. Por ejemplo, si
los sabores disponibles fueran 8 y la cantidad de dulces que Ana comprará es 100, el problema se
vuelve muchı́simo más difı́cil y el método anterior no funcionará bien.

Si se observa, este problema es distinto a todos los estudiados hasta el momento, y básicamente
se trata de buscar combinaciones de objetos, pero no todos los objetos son distintos (para el caso,
los dulces de fresa los consideramos todos iguales, los de menta también y los de limón también).
Hay una forma muy ingeniosa de resolver este problema, y por lo importante de la técnica, suele
dedicársele una sección.

Haremos lo siguiente: los 10 dulces los vamos a interpretar como 10 objetos iguales, 10 bolas por
ejemplo, y para distinguir cuáles son de cada sabor, incluiremos 2 “separadores”; luego, estos 12
objetos se permutan, m es la cantidad de bolas que quedan a la izquierda del primer separador, f
es la cantidad de bolas que quedan entre los separadores y l es la cantidad de bolas que quedan
a la derecha del segundo separador. Además, la cantidad de permutaciones con repetición con 10
12!
bolas y 2 separadores es P (10, 2) = = 66.
10!2!




|{z } | {z } | {z }
m f l

El problema general se resuelve de la misma forma:

39
4.2 MULTICOMBINATORIO 4 Extensiones del Número Combinatorio

Teorema 4.1. Dada una colección de objetos clasificados en k tipos de objetos (los objetos del
mismo tipo son iguales entre sı́, y distintos de cualquier objeto de otro tipo), el total de formas de
escoger n objetos es
(n + k − 1)!
P (n, k − 1) =
n!(k − 1)!

| | || | · · · |
Demostración. Se considera en principio que los n objetos son todos iguales, y para distribuir los
objetos entre las posibles k clases, se agregan k − 1 separadores. El total de configuraciones es igual
a las permutaciones con repetición P (n, k − 1).
Observe que la respuesta puede verse como un combinatorio también: en total, se tienen n + k − 1
espacios y se escogen los n (o bien los k − 1) en los que se ubican las bolas (o bien los separadores),
por lo que la cantidad de configuraciones buscadas es

n k−1
Cn+k−1 o bien Cn+k−1
Finalmente, una versión muy utilizada de separadores es la siguiente:

Teorema 4.2. El total de soluciones enteras no negativas de la ecuación x1 + x2 + · · · + xk = n


n
es Cn+k−1

La relación con el problema anterior es evidente, porque xi representa la cantidad de objetos del
tipo i; el particular, el problema de Ana es equivalente a resolver la ecuación x1 + x2 + x3 = 10,
con xi ∈ Z+ 0 . Note que hay configuraciones que tienen cero bolas de algún tipo, esto, en el esquema
de los separadores, se da cuando los separadores están juntos, o cuando un separador está a la
izquierda de todas las bolas o a la derecha de todas las bolas.

Otro detalle importante que comentar es que a veces se busca configuraciones que tengan al menos
uno de cada tipo, es decir, xi ≥ 1. En tal caso los separadores se ubican únicamente en los n − 1
espacios entre las n bolas, a lo sumo un separador por espacio; ası́, el total de configuraciones con
k−1
esta nueva restricción es Cn−1 .

4.2. MULTICOMBINATORIO
Dados n objetos distintos, el total de formas de escoger p de ellos es np ; esto, como ya se estu-


dió en reiteradas ocasiones, puede interpretarse con modelo de cajas: en la caja 1 se introducen
los p objetos sı́ escogidos, y en la caja 2 los n − p objetos no escogidos, ası́, este problema es
equivalente a: ¿De cuántas formas es posible distribuir n objetos distintos en dos cajas, tal que en
la primera caja siempre hayan p objetos? Lo interesante de este planteamiento es que se puede
extender fácilmente a más número de cajas; al número que obtendremos en ese caso se le llama
multicombinatorio, y no es más que una extensión y generalización del número combinatorio.

40
4.3 PROBLEMAS 4 Extensiones del Número Combinatorio

Definición 4.1. Multicombinatorio: Al total de formas de distribuir n objetos distintos en k cajas,


tal que en la primera caja siempre hayan x1 objetos, en la segunda caja x2 objetos, ..., en la k−ésima
caja xk objetos, con x1 +x2 +· · ·+xk = n, se le llama multicombinatorio de n escoger x1 , x2 , . . . , xk ,
y se denota  
n
x1 , x2 , . . . , xk
Es interesante que este problema puede resolverse sin mayores dificultades a partir del número
combinatorio estándar, lo cual se enuncia en el siguiente teorema

Teorema 4.3. El multicombinatorio de n escoger x1 , x2 , . . . , xk es:


  k  
n Y n − (x1 + x2 + · · · + xs−1 )
=
x1 , x2 , . . . , xk s=1
xs

Demostración. De los n objetos escogemos los x1 que se colocarán en la primera caja, lo cual se
puede hacer de xn1 formas; luego, escogemos los x2 objetos de entre los n − x1 objetos restantes,
que se colocarán en la segunda caja, lo cual es posible de n−x 1

x2
formas, y ası́ sucesivamente, hasta
que para la caja k−ésima se escogerán xk objetos de entre los n − (x1 + x2 + · · · + xk−1 ) = xk
objetos, lo cual se podrá hacer sólo de una forma, xxkk . Por lo tanto, el total de posibilidades es
     
n n − x1 n − (x1 + x2 ) n − (x1 + x2 + · · · + xk−1 )
···
x1 x2 x3 xk

Este número también tiene modelos equivalentes por caminos y por relaciones algebraicas. El
vı́nculo del multicombinatorio con expresiones algebraicas viene de desarrollar los multinomios,
n

por ejemplo (x + y + z)n , el coeficiente de xa y b z c (con a + b + c = n) es el multicombinatorio a,b,c ;
¡demuéstrelo!

Además, es importante que identifique el nexo entre separadores y el multicombinatorio, son pro-
blemas aparentemente iguales, pero son muy distintos; observe por ejemplo que el total de términos
del desarrollo del trinomio (x + y + z)n es igual a la cantidad de términos xa y b z c , y esto es igual a
2
la cantidad de soluciones de a + b + c = n, lo cual, por separadores es Cn+2 .

4.3. PROBLEMAS
1. Determine el número de formas que pueden ordenarse en un estante 4 libros distintos de
Combinatoria, 5 libros distintos de Geometrı́a, 3 libros distintos de Álgebra y 8 libros distintos
de Cálculo, si los de Geometrı́a deben estar siempre antes que los de Álgebra.

2. ¿En cuántas de las permutaciones del número 23814425 aparecen los dı́gitos impares en forma
creciente de izquierda a derecha?

3. Se han encargado 20 pupusas de entre los siguientes tipos: revueltas, de queso, de chicharrón,
de frijol con queso, de queso con loroco y de ayote. ¿De cuántas formas puede hacer la compra?

41
4.3 PROBLEMAS 4 Extensiones del Número Combinatorio

a) Si se tiene que llevar al menos 7 de queso.


b) Si se tiene que llevar a lo sumo 2 de chicharrón y 10 de ayote.
c) Si se tiene que llevar al menos 3 de cada clase.

4. Encuentre el número de secuencias no-decrecientes de largo 10

a1 ≤ a2 ≤ a3 ≤ · · · ≤ a10

donde ai ∈ {1, 2, 3, . . . , 100}

5. ¿Cuántas 11-combinaciones pueden formarse de las letras x, y, z si todas las letras deben
aparecer al menos dos veces y a lo sumo 5?

6. ¿Cuántas cadenas existen de 10 dı́gitos ternarios (0, 1 ó 2) que contengan exactamente dos
0, tres 1, y cinco 2?

7. ¿Cuántas soluciones existen para la desigualdad

x1 + x2 + x3 ≤ 11

donde x1 , x2 y x3 son enteros no negativos?

8. ¿De cuántas formas pueden ordenarse n ceros y k − 1 unos si no hay dos 1 consecutivos?

9. Se tiran 12 dados idénticos al aire, ¿cuál es la probabilidad de que uno de los números
1, 2, . . . , 6 no aparezca?

10. ¿De cuántas formas puede distribuirse 20 bolas iguales en 6 cajas, de tal forma que en la
primera caja hay al menos 4 bolas y en la ultima caja no más de 5?

11. Existen 5 formas de expresar el número 4 como suma de dos enteros no negativos tomando
en cuenta el orden: 4 = 0 + 4 = 1 + 3 = 2 + 2 = 3 + 1 = 4 + 0. Dados los naturales r y n,
determine:

a) El número de formas de expresar 200 en r sumandos.


b) El número de formas de expresar n en 200 sumandos.
c) El número de formas de expresar n en r sumandos tales que todos sean mayores o iguales
que 5.

12. José comprará 20 galletas de distintos sabores: fresa, chocolate, vainilla y limón. ¿De cuántas
formas puede hacer la compra si tiene que llevar un número de galletas de vainilla que sea 4
veces el número de galletas de limón?

13. Determine el número de soluciones enteras no negativas de 3x1 + 5x2 + x3 + x4 + x5 = 20.

14. Determine el número de términos de la expansión de (x1 + x2 + x3 + · · · + xn−1 )n−1 .

15. Determine el número de soluciones enteras no negativas de rx1 + x2 + x3 + · · · + xn = kr.

16. En la expansión de (a + b + c + d)48

42
4.3 PROBLEMAS 4 Extensiones del Número Combinatorio

17. Encuentre el número de enteros positivos x tales que x ≤ 9999999 y la suma de sus dı́gitos
sea igual a 31.

18. Si Lorena tira un dado cinco veces, ¿cuál es la probabilidad de que la sumas de sus cinco
tiradas sea 20?

19. ¿Cuántas soluciones hay, entre 1 y 9 inclusive, de la ecuación x1 + x2 + x3 + x4 = 26?

20. Encuentre el número de soluciones enteras positivas de la ecuación:

(x1 + x2 + x3 )(y1 + y2 + y3 + y4 ) = 77

a) Encontrar el coeficiente de a8 b10 c15 d15 .


b) Determinar el número de términos de la expansión.
6
21. Determine el coeficiente de x5 en la expansión de (1 + x + x2 + · · · + x1000 ) .

43
5 Principios de Conteo 2

5. Principios de Conteo 2
5.1. PRINCIPIO DE INCLUSIÓN - EXCLUSIÓN
El principio, tal comos se describió en el primer capı́tulo, puede ser extendido extendido de forma
natural al caso de la reunión de n conjuntos. La forma que adopta el principio en este caso es:
Teorema 5.1. Principio de Inclusión - Exclusión: Dados n conjuntos A1 , A2 , . . . , An , el cardinal
de la unión está dado por


[n X X X
Ai = |Ai | − |Ai ∩ Aj | + |Ai ∩ Aj ∩ Ak | + · · · + (−1)n+1 |A1 ∩ A2 ∩ · · · ∩ An |



i=1 i i<j i<j<k

La fórmula pretende asegurar que todos los elementos son contados una y sólo una vez, lo que
puede ser verificado después de un momento de reflexión.

5.2. DESÓRDENES

EJEMPLO 5.1
Si consideramos el orden de los números naturales 1, 2, 3, 4, en la permutación 3142 ningún ele-
mento está en su posición natural. A una permutación con tal propiedad le denominaremos un
desorden o un desarreglo. ¿De los 24 posibles ordenamientos de tales números, cuántos desórdenes
existen?

Denotemos por D4 tal número de desórdenes; por F1 el número de las permutaciones que dejan
fijo un elemento en su posición; por F2 las que dejan en su posición dos de los 4 elementos, por F3
las que dejan en su puesto tres de los 4 elementos; de manera completamente similar definiremos
F4 . Ası́, por el principio de inclusión-exclusión (5.1) tenemos el siguiente resultado:

D4 = 4! − F1 + F2 − F3 + F4
Ahora bien, F1 se descompone en los que dejan fijo el 1 en su posición, que son 6, los que dejan
fijo el 2, que son otras 6; hay 6 que dejan el 3 en su posición y 6 que dejan fijo el 4; ası́, F1 es 24.
De las que dejan fijos dos de los cuatro elementos están los que dejan fijos el 1 y 2, los que dejan
fijos el 1 y 3, el 1 y 4; los que dejan fijos el 2 y 3, el 2 y 4; finalmente tenemos los que dejan fijos el
3 y el 4. Como en cada uno de estos caso, que son seis, el número de permutaciones es 2, resulta
que F2 es 12. El caso de que dejen fijos tres elementos contiene los casos siguientes: que dejen fijos
1, 2, 3; 1, 2, 4; 1, 3, 4 y el caso que dejen fijos 2, 3, 4; en total son cuatro casos y cada uno de ellos
tiene una permutación por lo que F3 es igual a 4. Finalmente, F4 contiene una única permutación,
es decir F4 es 1. En resumen tenemos:

D4 = 24 − 24 + 12 − 4 + 1 = 9
Hay en consecuencia 9 desórdenes en las permutaciones de orden 4.

Esta claro que esta relación se puede generalizar.

44
5.3 RECURRENCIA 5 Principios de Conteo 2

Teorema 5.2. El total de desórdenes de orden n, denotado por Dn , es


n  
X n
k
Dn = (−1) (n − k)!
k=0
k

Demostración. El total de permutaciones es Pn = n!. Tomando la misma notación del ejemplo


anterior, Fk denota aquellas permutaciones que tienen a k (al menos) de sus elementos en la posición
que les corresponde; ası́, el total de de permutaciones de Fk lo contamos primero escogiendo los k
que quedarán en la posición que les corresponde, lo cuál es posible hacerlo de Cnk formas, y luego
permutando los restantes n − k objetos, lo cual es posible hacerlo de Pn−k = (n − k)! formas, y por
el principio de la multiplicación, Fk = Cnk (n − k)!. Luego, por el principio de inclusión-exclusión:

Dn = n! − F1 + F2 − F3 + · · · + (−1)k Fk + · · · + (−1)n Fn
= (−1)0 Cn0 (n − 0)! + (−1)1 Cn1 (n − 1)! + (−1)2 Cn2 (n − 2)! + · · · + (−1)n Cnn (n − n)!
n  
k n
X
= (−1) (n − k)!
k=0
k

Observe que esa expresión se puede manipular algebraicamente, y reescribirse como


n
X (−1)k
Dn = n!
k=0
k!

5.3. RECURRENCIA

EJEMPLO 5.2
La Torre de Hanoi:
Se dispone de n discos de diferentes medidas y de tres clavijas en donde éstos pueden ser colocados.
Inicialmente todos los discos están colocados en una de las clavijas y ordenados de abajo hacia
arriba de mayor a menor. El problema consiste en trasladarlos a otra de las clavijas siguiendo las
reglas siguientes:

i) Los discos se mueven uno por uno.

ii) En ningún caso puede colocarse un disco sobre un disco de radio menor.

Obsérvese que para cumplir con la condición segunda resulta indispensable disponer de tres clavi-
jas. El problema consiste en determinar el número mı́nimo de movimientos requeridos para pasar

45
5.3 RECURRENCIA 5 Principios de Conteo 2

todos los discos de una clavija a otra.

Sea mn el número mı́nimo de movimientos para pasar n discos de una clavija a otra. Es obvio que
para poder mover el disco que se encuentra en el fondo, debemos pasar de una clavija a otra los n−1
discos menores a otra clavija, lo que requiere mn−1 como número mı́nimo de movimientos. Hecho lo
anterior, debemos pasar el disco mayor a la tercera clavija, lo que exige un movimiento, y finalmente
pasar los n−1 discos menores a esta última clavija, lo que por definición requiere nuevamente mn−1
movimientos como mı́nimo. Ası́ en total se requieren como mı́nimo mn = 2mn−1 + 1 movimientos.
Por otra parte, cuando se tiene sólo 1 disco, obviamente, el número mı́nimo de movimientos es
m1 = 1; es decir: 
mn = 2mn−1 + 1
m1 = 1
Información suficiente para calcular el valor de mn para los diferentes valores de n.

EJEMPLO 5.3
La Sucesión de Fibonacci:

Suponemos que cada mes la hembra de una pareja de conejos pare una pareja de conejos (de dife-
rente sexo). Dos meses más tarde la hembra de la nueva pareja pare una nueva pareja. Determinar
el número de parejas de conejos al finalizar el primer año, si se supone que al inicio sólo se dispone
de una pareja de conejos en edad reproductiva y que en el perı́odo en mención no hay defunciones.

Por ejemplo, al final del primer mes tendremos dos parejas, al final del segundo mes se tendrán
tres parejas. Denotemos por Fn el número de parejas al final del enésimo mes. Este número de
parejas debe ser igual al número de parejas en el mes n − 1, que es igual a Fn−1 , más los recién
nacidos, cuyos padres únicamente pueden ser las parejas existentes en el mes n − 2. Ası́, resulta la
relación:

 Fn = Fn−1 + Fn−2
F0 = 1
F1 = 2

De nuevo, con estas relaciones podemos calcular el valor de Fn para los diferentes valores de n. Con
ella se logra la conocida como sucesión de Fibonacci: 1, 2, 3, 5, 8, 13, 21, 34, 55, . . .. El problema de
determinar el número de parejas de conejos al final del primer año es simplemente el de calcular
F12 , que es igual a 377.

EJEMPLO 5.4
El Modelo de Reproducción de Conejos y las Cadenas de ceros y unos que no tienen dos unos
seguidos.

Veremos como este problema del número de parejas de conejos al final del enésimo mes puede
ponerse en correspondencia con el problema de las cadenas de ceros y unos de longitud n que no
tienen dos unos seguidos.

46
5.4 PRINCIPIO DE CASILLAS 5 Principios de Conteo 2

Suponga una cadena de ceros y unos tiene la propiedad de no tener dos unos consecutivos. Cada 1
en la cadena representará el nacimiento de una pareja; cada pareja de conejos, se identificará con
el último 1 en la cadena; los demás unos de la cadena que aparecen antes, representarán el árbol
genealógico de la pareja que representa la cadena. La cadena de sólo ceros estará asociada a la pa-
reja original. La restricción de que las cadenas no tengan dos 1 consecutivos es justamente debido
a que en el modelo cada pareja sólo puede comenzar a procrear al finalizar el segundo mes.
Por ejemplo, la cadena 00101001 es el documento de identidad de la pareja de conejos que nació al
finalizar el octavo mes, cuyos padres nacieron en el quinto mes; sus abuelos nacieron al finalizar el
tercer mes y sus bisabuelos la pareja de conejos original.

Es evidente que esta correspondencia es biunı́voca; a cada cadena de ceros y unos con la propiedad
de no tener dos unos seguidos, le corresponde una sola pareja de conejos y a cada pareja de conejos,
le corresponde una sola cadena.

Podemos ahora, apoyados en la correspondencia anterior, proponernos contar el número de parejas


de conejos, mediante el conteo de las cadenas de ceros y unos con la propiedad de no tener dos
unos seguidos. Sólo con el propósito de ilustrar nos restringiremos al caso de cadenas de longi-
tud 6, el caso general puede ser analizado de forma completamente análoga. El problema es ası́ el
de contar las cadenas de longitud 6 de ceros y unos con la propiedad de no tener dos unos seguidos.

Obsérvese primero que para que no hayan dos unos seguidos se requiere que el número de unos sea
menor o igual a tres, de lo contrario no se dispondrá de suficientes ceros para separarlos. Podemos
entonces clasificar nuestras cadenas según el número de unos que posea: Cadenas que no tienen
unos; los 6 son ceros y los espacios disponibles para colocar los unos, son 7. Se tiene entonces C70
en este caso. Cadenas que tienen exactamente un uno. Obviamente la cadena debe tener 5 ceros,
lo que permite disponer de 6 espacios para colocar el uno, en este caso el número de cadenas debe
ser C61 . He aquı́ la lista de todas estas cadenas: 100000, 010000, 001000, 000100, 000010, 000001.
Cadenas que tienen exactamente dos unos. En este caso deben haber cuatro ceros, lo que permite
disponer de 5 espacios para colocar los 2 unos. El total deben ser C52 cadenas, que son los siguien-
tes: 101000, 100100, 100010, 100001, 010100, 010010, 010001, 001010, 001001, 000101. Cadenas que
tienen tres unos. Por supuesto la cadena tiene tres ceros y los espacios disponibles para colocar
los tres unos, son 4; por lo tanto tendremos C43 cadenas; estas son: 101010, 101001, 100101, 010101.

El total de cadenas con la propiedad será entonces la suma: 1 + 6 + 10 + 4 = 21, número que
corresponde a F6 en la sucesión de Fibonacci.

5.4. PRINCIPIO DE CASILLAS


El principio de casillas es posiblemente uno de los teoremas más evidentes, y cualquiera podrı́a
pensar que no tiene utilidad alguna una aseveración tan evidente, sin embargo, en los problemas
que se plantean en este capı́tulo se puede apreciar lo poco evidente que resultan muchos problemas
cuya clave radica en el principio de casillas.

EJEMPLO 5.5
Una recta no puede cortar internamente a los tres lados de un triángulo simultáneamente.

47
5.5 PROBLEMAS 5 Principios de Conteo 2

En este caso hay que crear tanto las cajas como los objetos; digamos que se traza la recta L,10 la
cual genera dos semiplanos, éstos serán las cajas; por otra parte, los vértices del triángulo serán
los objetos, que son en tres en total. Por el principio de casillas, hay un semiplano que tiene al
menos dos vértices, por lo tanto, la recta L no corta al lado definido por esos vértices.

5.5. PROBLEMAS
1. ¿Cuántos enteros entre 1 y 1000, incluyéndolos, no son divisibles entre 2, 3 ó 5?

2. ¿Cuántos números naturales menores o iguales que 10000 son múltiplos de 4, 5 ó 7? ¿Cuántos
son múltiplos de 4, 10 ó 14?

3. En cierta escuela hay 100 alumnos. De ellos, 50 saben inglés, 30 saben alemán y 30 saben
francés. Además, 10 saben inglés y francés, 14 saben francés y alemán, 11 saben inglés y
alemán, y 6 saben los tres idiomas. Determinar cuántos alumnos no saben ninguno de los
tres idiomas.

4. Un grupo de 102 estudiantes se examinan en Matemáticas, Sociales y Lenguaje. De entre


ellos, 92 pasaron Matemáticas, 75 Sociales y 63 Lenguaje, 65 pasaron Matemáticas y Sociales,
54 Matemáticas y Lenguaje y 48 Sociales y Lenguaje. ¿Cuántos estudiantes pasaron las tres
materias?

5. ¿Cuántos números del 1 al 1000000 no son ni cuadrados perfectos, ni cubos perfectos, ni


potencias cuartas perfectas?

6. En un grupo de 100 indios hay 40 que hablan hindi, 40 que hablan bengalı́ y 20 que hablan
penjabi. Hay 20 que hablan hindi y bengalı́, y 5 que hablan hindi y penjabi. Hay 31 que
hablan al menos dos de las tres lenguas y 33 que no hablan ninguna de ellas. ¿Cuántos
hablan las tres lenguas?

7. ¿Cuántos números menores que 400 tienen la propiedad de no tener divisor alguno en el
conjunto {6, 10, 15}?

8. ¿Cuántos números naturales menores que 1000 tienen la propiedad de ser divisibles por 12,
pero no divisibles ni por 5, ni por 7?

9. De todas las combinaciones posibles de 5 elementos del conjunto {1, 2, . . . , 10} ¿Cuántas no
incluyen ni el número 8, ni el número 9?

10. Determine el número de desórdenes posibles del conjunto {1, 2, 3, 4, 5}.

11. Si se asume el orden natural en el conjunto {1, 2, 3, 4, 5}, ¿cuántas permutaciones dejan fijos
en su posición exactamente a dos de los cinco números?
10
Como se busca cortar internamente a los lados del triángulo, suponemos que L no coincide con ninguno de los
lados ni pasa por algún vértice.

48
5.5 PROBLEMAS 5 Principios de Conteo 2

12. Considere los números naturales menores o iguales a 100 en su descomposición en factores
primos. ¿En cuántos de ellos no hay un factor primo repetido?

13. Un año es bisiesto cuando:

i) Es múltiplo de 4 pero no de 100, o


ii) Es múltiplo de 400.

Por ejemplo, 1600 y 1924 fueron años bisiestos, mientras que 2200 no lo será. Encuentre el
número total de años bisiestos entre los años 1000 y 3000.

14. ¿Cuántas permutaciones de 1234 son tales que el 1 no está en la primera posición, el 2 no
está en la segunda posición, el 3 no está en la tercera posición, y el 4 no está en la cuarta
posición?

15. En una oficina hay 10 empleados. Cada uno es especialista en una labor distinta a la de los
demás. Para no aburrirse, les gusta intercambiar sus puestos; sin embargo, el buen funciona-
miento de la oficina exige que en cada momento haya exactamente 4 empleados trabajando
en su especialidad. ¿Cuántas distribuciones de los puestos se pueden hacer bajo estas condi-
ciones?

16. Se sabe que la clave de acceso de una comutadora es una permutación de CLAVE2654.
Sin embargo, como el dueño sufre de dislexia, programó en su compuradora la siguiente
regla: Para conseguir acceso, es suficiente introducir 5 caracteres en la posición correcta, los
caracteres restantes pueden estar en la posición correcta o no. ¿De cuántas formas puede
acceder a la computadora?

17. Ana quiere comprar 15 dulces, los sabores disponibles son: coco, vainilla, fresa y canela. Como
a su mamá le gustan los dulces de coco, piensa llevar al menos 5 de este sabor; además, nunca
ha probado los dulces de canela, por lo que llevará a lo sumo 4 de este sabor. ¿De cuántas
formas puede Ana comprar los dulces?

18. Para comprar en una carnicerı́a cada cliente toma un número (los números van saliendo
ordenados 1, 2, . . . , n), pero el carnicero estaba de mal humor un dı́a y decidió atender a las
n personas que esperaban su turno de una manera muy extraña. ¿De cuántas formas puede
atenderlos si los que tienen número impar serán atendidos en su turno, mientras que los que
tienen número par no?

19. En un grupo de 102 estudiantes se examinan en Matemáticas, Sociales y Lenguajes. De entre


ellos, 92 pasaron Matemáticas, 75 Sociales y 63 Lenguaje, 65 pasaron Matemáticas y Sociales,
54 Matemáticas y Lenguaje, y 48 Sociales y Lenguaje. ¿Cuántos estudiantes pasaron las tres
materias?

20. Para rendir un examen, 4 alumnos se sientan en una fila de 10 asientos. ¿De cuántas maneras
pueden ubicarse, si no puede haber alumnos sentados contiguamente?

21. Siete libros diferentes, tres de Historia, dos de Matemática y dos de Quı́mica, son colocados
en el estante de una biblioteca. ¿En cuántas de las posibles formas de ubicarlos no aparecerán
todos los libros de una misma materia?

49
5.5 PROBLEMAS 5 Principios de Conteo 2

22. ¿De cuántas formas se puede colocar tres x, tres y y tres z de modo que no aparezcan la
misma letra tres veces consecutivas?
23. En cierto ecosistema hay 18 especies de animales. Cada especie depredadora caza 2 especies
diferentes. A su vez cada especie no depredadora es perseguida por 3 especies diferentes.
Además, se sabe que toda especie es perseguida o depredadora, y ninguna de las dos cosas a
la vez. ¿Cuántas especies depredadoras hay?
24. Consideremos una cuadricula de n × n ¿cuántos cuadrados tales que apoyen alguno de sus
lados en el borde inferior o el borde izquierdo de de la cuadricula?
25. Consideremos una cuadricula de 8 × 10 casillas. ¿Cuántos rectángulos pueden marcarse en
ella que tengan por lo menos uno de sus lados apoyado en algún borde?
26. ¿Cuántas soluciones enteras tiene la ecuación x1 + x2 + x3 = 28 si las variables están sujetas
a la condición: 3 ≤ x1 ≤ 9, 0 ≤ x2 ≤ 8, 7 ≤ x3 ≤ 17?
27. ¿Cuántas soluciones enteras tiene la ecuación x1 + x2 + x3 + x4 = 20 si las variables están
sujetas a la condición x1 ≤ 6, x2 ≤ 7, x3 ≤ 8, x4 ≤ 9?
28. Hallar el número de soluciones enteras de la ecuación x1 + x2 + x3 + x4 + x5 + x6 = 20 que
satisfacen las condiciones: 1 ≤ x1 ≤ 6, 1 ≤ x2 ≤ 7, 3 ≤ x3 ≤ 9, 4 ≤ x4 ≤ 11.
29. ¿De cuántas formas se pueden distribuir 10 premios distintos entre 4 estudiantes de modo
que exactamente dos estudiantes no reciban ninguno? ¿De cuántas formas puede hacerse esto
de modo que al menos dos estudiantes no reciban premio?
30. Demuestre mediante la definición de coeficiente multinomial que:
  X m  
n n−1
= (1)
n1 , n2 , . . . , nm i=1
n1 , . . . , ni − 1, ni + 1, . . . , nm

31. Demuestre que


X n

= mn
n1 , n2 , · · · , nm
donde el sumatorio contempla todas las soluciones enteras no negativas de n1 +n2 +· · · nm = n
32. Sea k, n, r ∈ N. Muestre que el número de soluciones enteras de la ecuación

x1 + x2 + x3 + · · · + xn = r

tal que 0 ≤ xi ≤ k para cada i = 1, 2, . . . , n está dado por


n   
X
i n r − (k + 1)i + n − 1
(−1)
i=0
i n−1

33. Determinar el número de regiones determinadas por n rectas en el plano.


34. Determine el número de cadenas de ceros y unos que tienen la propiedad de no tener dos
unos consecutivos.

50
5.5 PROBLEMAS 5 Principios de Conteo 2

35. Supongamos que se desea cubrir un rectángulo de dimensión n × 2 con rectángulos de 2 × 1.


¿De cuántas formas es posible hacerlo?

36. ¿De cuántas formas es posible descomponer un número natural como suma sólo de los natu-
rales 1 y 2?

37. Llamamos triangulación de un polı́gono a su descomposición en triángulos cuyos lados son


ya sean diagonales o lados del polı́gono, de forma tal que el polı́gono queda completamente
cubierto, sin superposiciones.

a) Demuestre que, independiente de la triangulación, el número de triángulos utilizados en


la descomposición es el mismo e igual a n − 2.
b) Demuestre que el número de diagonales utilizadas en cualquier triangulación es siempre
n − 3.
c) Determinar el número de posibles triangulaciones de un polı́gono de n lados.

38. Considere un triángulo equilátero cuyo lado tiene longitud n. Se triángula trazando rectas
paralelas a los lados que cortan segmentos de 1 a los lados del triángulo; se forman triángulos
equiláteros de diferentes longitudes de lado. Determine el número total de tales triángulos.

39. Se dispone de un número ilimitado de monedas de las denominaciones de 1,5,10 y 25 centavos


de dólar. ¿De cuántas formas se puede formar un total de 30 centavos? Determine una relación
de recurrencia que permita calcular el número de posibilidades de cambiar una cantidad
cualquiera de dinero.

40. Sean C un conjunto con 2n números reales, n ≥ 1 y an el número de comparaciones que


deben efectuarse entre los elementos de C para determinar el máximo y el mı́nimo de C.
Encontrar una relación de recurrencia para calcular an y resolverla.

41. Tenemos cinco puntos con coordenadas enteras en el plano cartesiano. Probar que si sacamos
los puntos medios para cada par de estos puntos, existirá un punto medio cuyas coordenadas
serán también enteras.

42. Una prueba de concurso posee diez preguntas de selección múltiple, con cinco alternativas
cada una. ¿Cuál es el número mı́nimo de candidatos que deberı́an hacer el examen para
garantizar que por lo menos dos de ellos tendrán las mismas respuestas para todas las pre-
guntas?

43. Dados 6 puntos sobre una circunferencia, coloreamos de azul o verde todos los segmentos que
ellos determinan. Demuestre que entre todos los triángulos que quedan formados, hay por lo
menos uno cromático, es decir, un triángulo con sus tres lados del mismo color.

44. Demuestre que si con 2 colores se pintan las diagonales de un pentágono regular, siempre
hay un vértice del que salen dos diagonales del mismo color.

45. Para una reunión cientı́fica se han contratado 5 traductores, que deberán cubrir 6 lenguas
diferentes. Si cada una de éstas requiere el empleo de 3 traductores, demostrar que alguno
de los intérpretes deberá traducir por lo menos 4 idiomas.

51
5.5 PROBLEMAS 5 Principios de Conteo 2

46. En un cajón hay calcetines negros, rojos, azules y blancos. ¿Cuál es el menor número de
calcetines que hay que sacar para estar seguros de que hay al menos dos del mismo color?

47. En un estadio hay diez mil personas. Demostrar que hay al menos un grupo de 28 personas
que nacieron el mismo dı́a.

48. ¿Cuál es el mayor número de reyes que pueden ser colocados en un tablero de ajedrez de
manera que ninguno dé jaque a ningún otro?

49. A un estadio de fútbol han asistido 37000 espectadores. ¿Cuántos de ellos, como mı́nimo,
cumplen años el mismo dı́a?

50. Hay 100 personas sentadas en una mesa circular a distancia constante entre sı́ y al menos 51 de
ellas son mujeres. Verificar que hay al menos 2 mujeres sentados en posiciones diametralmente
opuestas.

51. Considere los primos 2, 3, 5. El conjunto A está formado por los números naturales que se
generan multiplicando distintas potencias de estos primos, es decir, los números de la forma

2α 3β 5γ

con α, β, γ enteros no negativos. Demuestre que de cualquier escogitación de 9 números de


A siempre hay 2 que al multiplicarlos generan un cuadrado perfecto.

52. Suponga que los números del 1 al 10 están ubicados en algún orden sobre una circunferencia.
Prueba que alguna suma de tres números consecutivos suma 17.

53. En un planeta llamado Ω (omega), más de la mitad de la superficie es tierra firme. Probar
que en Ω se podrı́a excavar un túnel recto a través del centro del planeta, comenzando y
finalizando en tierra firme.

54. ¿Cuántas veces, como mı́nimo, debe lanzarse un par de dados para asegurarse que el puntaje
obtenido (la suma de los dados) se repita?

55. Sean a, b, c, d enteros, demuestre que (a − b)(b − c)(c − d)(d − a)(a − c)(b − d) es múltiplo de
12.

56. Una prueba de aptitud posee 10 preguntas de selección múltiple, con cinco alternativas cada
una. ¿Cuál debe ser el mı́nimo número de alumnos que deben dar la prueba (sin dejar
respuestas vacı́as) para el cual podamos garantizar que por lo menos dos de ellos tendrán
exactamente las mismas respuestas para todas las preguntas?

57. En una caja hay 10 libros en francés, 20 en castellano, 8 en alemán, 15 en ruso y 25 en


italiano. ¿Cuantos debo sacar para estar seguro de que tengo 12 en un mismo idioma?

58. En un bar hay 95 mesas y un total de 465 sillas. ¿Podemos asegurar que hay una mesa con
6 sillas?

59. Se tiene un conjunto de diez números naturales. Demostrar que hay al menos un par cuya
diferencia es múltiplo de 9.

52
5.5 PROBLEMAS 5 Principios de Conteo 2

60. Demuestre que si del subconjunto de números naturales 1, 2, . . . , 10 extraemos seis números,
con seguridad habrá dos que suman 11.

61. De los números 1, 2, . . . , 100 se toman 51. Demuestre que de estos, hay una pareja que son
primos relativos, y que hay otra pareja tal que uno divide al otro y el cociente es potencia
de 2.

62. Sean a1 , a2 , . . . , a100 y b1 , b2 , . . . , b100 dos permutaciones de 1, 2, 3, . . . , 100. Demuestra que,


entre los productos
a1 b1 , . . . , ai bj , . . . , a100 b100
hay dos con el mismo residuo al dividirse entre 100.

63. Los números 1, 2, . . . , 9 se dividen en tres grupos. Probar que el producto de los números en
uno de dichos grupos, sean cual sean estos, siempre debe ser mayor de 71.

64. En un cubo de lado 10 se colocan 999 puntos. ¿Es posible encontrar siempre un cubo de
lado 1 dentro del cubo de lado 10 que no contenga alguno de los puntos?

65. Con los vértices de una cuadrı́cula de 6 × 9 se forman 24 triángulos. Demuestre que hay 2
triángulos que tienen un vértice en común.

66. Las entradas de una matriz 3 × 3 son los números 0, 1, −1. Probar que entre las ocho sumas
que se obtienen por filas, columnas y diagonales, hay dos iguales.

67. Se tienen los números 1, 2, . . . , 2n escritos en una pizarra. Se tachan n–1 de ellos. Probar que
entre los números que quedaron sin tachar en la pizarra, hay al menos dos de ellos que son
consecutivos.

68. En una pizarra se escriben los números 1, 2, . . . , 2n, probar que si se eligen aleatoriamente
n+1 números de entre ellos, entonces entre los elegidos habrá un par de modo que uno divide
al otro.

69. Dados 27 números impares positivos menores que 100, demostrar que hay al menos dos de
ellos cuya suma es 102.

70. 17 personas se comunican por correo, enviando cada persona una carta a cada una de las
demás. En las cartas sólo son discutidas tres temáticas distintas. Las cartas enviadas mutua-
mente entre dos personas tratan ambas a cerca de una sola de esas temáticas, dado que una
persona envı́a una carta y la persona a la que va dirigida le responde. Pruebe que hay un
grupo de al menos 3 personas tales que en todas las cartas que se enviaron entre sı́ discutieron
a cerca de la misma temática.

71. Algunos de los cuadritos de una cuadricula de 3 × 7 se pintan de negro y los otros se dejan en
blanco. Probar que forzosamente las lı́neas de la cuadrı́cula forman un rectángulo en cuyas
cuatro esquinas los cuadraditos tienen el mismo color (los cuatro blancos o los cuatro negros).

72. Dado un cuadrado de diagonal 3, se marcan al azar 10 puntos. Demostrar que siempre
podemos encontrar al menos dos puntos que están a una distancia no mayor a 1.

53
5.5 PROBLEMAS 5 Principios de Conteo 2

73. En un triángulo de área 4 se colocan nueve puntos. Muestre que hay tres de ellos que forman
un triángulo de área menor o igual a 1.

74. Demuestre que un triángulo equilátero de lado 1 no puede ser cubierto totalmente con dos
triángulos equiláteros de lados menores que 1.

75. Un disco cerrado de radio 1cm contiene 7 puntos tales que todas las distancias entre dos de
ellos son mayores o iguales que 1, pruebe que uno de los 7 puntos es el centro del disco.

76. Nos damos un conjunto X de 10 números naturales distintos comprendidos entre 10 y 99.
Demostrar que hay dos subconjuntos distintos de X tales que la suma de los elementos de
los dos subconjuntos dan el mismo resultado.

77. En un segmento I de longitud 10, inicialmente blanco, se han marcado 10 segmentos disjuntos
con color rojo. Si no hay dos puntos en I a distancia 1 y ambos de color rojo, probar que la
suma de las longitudes de los intervalos es a lo sumo 5.

78. Se dispone de 100 tarjetas, numeradas del 100 al 199. El valor de cada tarjeta es la suma de
los dı́gitos que aparecen en ella. ¿Cuál es el numero mı́nimo de tarjetas que hay que extraer
para asegurar que haya tres con el mismo valor?

79. Demostrar que en una fiesta siempre hay dos personas que conocen al mismo número de
personas.

80. Comprobar que en una reunión de 6 personas siempre pasa que 3 de ellas se conocen entre
sı́ o bien 3 de ellas no se conocen entre sı́.

81. En una ceremonia de premiación, n personas se saludaron entre sı́ estrechándose las ma-
nos. Prueba que durante la ceremonia hubo siempre dos personas que estrecharon el mismo
número de manos.

82. En una reunión hay 201 personas de cinco nacionalidades diferentes. Se sabe que en cada
grupo de seis, al menos dos tienen la misma edad. Demuestra que hay al menos cinco personas
del mismo paı́s, de la misma edad y del mismo sexo.

83. Demuestre las siguientes identidades albegraicas:

a)
n
X n(n + 1)
i = 1 + 2 + 3 + ··· + n =
i=1
2

b)
n
X n(n + 1)(n + 2)
i(i + 1) = 1 · 2 + 2 · 3 + 3 · 4 + · · · + n(n + 1) =
i=1
3

c)
n
X n(n + 1)(n + 2)(n + 3)
i(i+1)(i+2) = 1·2·3+2·3·4+3·4·5+· · ·+n(n+1)(n+2) =
i=1
4

54
5.5 PROBLEMAS 5 Principios de Conteo 2

d) Exprese una identidad algebraica que generalice las identidades anteriores y demuéstrela.

84. Utilizando las identidades anteriores, demuestre

a)
n
X n(n + 1)(2n + 1)
i2 = 12 + 22 + 32 + · · · + n2 =
i=1
6

b)
n n
!2
X n2 (n + 1)2 X
i3 = 13 + 23 + 33 + · · · + n3 = = i
i=1
4 i=1

c) Encuentre una identidad algebraica para la suma de las potencias cuartas de 1 a n.

85. Sea S = {1, 2, 3, . . . , n + 1} en donde n ≥ 2, y sea T = {(x, y, z) ∈ S 3 |x < z, y < z}. Al


contar |T | de dos modos distintos, muestre que:
n    
X
2 n+1 n+1
k = |T | = +2
k=1
2 3

(3n)!
86. Demuestre que es entero para todo natural n.
2n 3n
87. Sea A = {1, 2, 3, . . . , n}, donde n ∈ N

a) Para k ∈ A, muestre que el número de subconjuntos de A en los que k es el número


mayor, es igual a 2k−1 .
b) Muestre a partir de lo anterior que:
n−1
X
2i = 2n − 1. (2)
i=0

55

You might also like